<<

VAJIRAM & RAVI

LIVE Test – 3 4th April 2020

Q1. With reference to the snow leopard, consider the following statements:

1. In 2017, status of Snow leopard in the Red List of the IUCN was changed from ‘endangered’ to ‘vulnerable’.

2. Project snow leopard was launched in 2009 in all hilly states in .

Which of the statements given above is/are correct? a. 1 only b. 2 only c. Both 1 and 2 d. Neither 1 nor 2

Answer: a

Explanation:

Statement 1 is correct.

Snow leopard is a member of the genus ‘Panthera’ to which tiger, leopard and lion also belong. Its Scientific name is “Panthera uncia”. They inhabit alpine and subalpine zones at elevations from 3,000 to 4,500 m (9,800 to 14,800 ft). It is native to the mountain ranges of Central and South Asia. The 12 range countries are Afghanistan, Bhutan, China, India, Kazakhstan, Kyrgyz Republic, Mongolia, Nepal, Pakistan, Russia, Tajikistan and Uzbekistan. India is home to 10% of the global population in less than 5% of its global range.

The major threat to snow leopard populations is poaching and illegal trade of skins and body parts. Poaching is linked to prey declines and livestock depredation. Greenhouse gas emissions will likely cause a shift of the treeline in the Himalayas and a shrinking of the alpine zone, which may reduce snow leopard habitat causing further danger.

In 2017, status of Snow leopard in the Red List of the IUCN was changed from ‘endangered’ to ‘vulnerable’.

• Endangered Species: According to IUCN, species are considered ‘endangered,’ if they are fewer than 2,500 and experiencing high rate of decline. • Vulnerable Species: Species are fewer than 10,000 and its population has declined at least 10% over three generations.

It is also listed in Schedule I of Wildlife (Protection) Act 1972 and Listed in Appendix I of the CITES, which makes trading of animal body parts (i.e., fur, bones and meat) illegal in signatory countries. VAJIRAM & RAVI

Statement 2 is incorrect.

Project snow leopard was launched in 2009 in the 5 Himalayan states of Jammu & Kashmir, HP, , and Arunachal Pradesh to conserve India’s high altitude wildlife populations such as Snow Leopard, Asiatic Ibex, Tibetan Argali, Urial, Chiru, Takin etc.

Source: https://vajiramias.com/current-affairs/india-plans-first-ever-snow-leopard- survey/5db250f61d5def7eb9248deb/

Q2. Which of the following pairs is/are correctly matched?

Select the correct answer using the code given below: a. 2 only b. 1 and 2 only c. 2 and 3 only d. 1, 2 and 3

Answer: a

Explanation:

Pair (1) is not correctly matched: NITI Aayog with Institute for Competitiveness as the knowledge partner released the India Innovation Index (III) 2019. The study examines the innovation ecosystem of Indian states and union territories. The aim is to create a holistic tool which can be used by policymakers across the country to identify the challenges to be addressed and strengths to build on when designing the economic growth policies for their regions. The states have been bifurcated into three categories: Major States, North-East & Hill states, and Union territories/City States/Small States. is the most innovative major state in India.

Pair (2) is correctly matched: School Education Quality Index (SEQI) was released by Niti Aayog. bagged the top spot on the index. SEQI has been developed by NITI Aayog through a collaborative process, including key stakeholders such as Ministry of Human Resource and Development (MHRD) and VAJIRAM & RAVI

World Bank. The index aims to evaluate the performance of States and UTs in the school education sector.

Pair (3) is not correctly matched: The Global Hunger Index (GHI) was brought out by US based International Food Policy Research Institute (IFPRI) in collaboration with Germany based Welthungerhilfe. Now IFPRI has stepped aside and Concern Worldwide has joined as a partner. It is published since 2000; this year’s report is the 14th one. The latest Global Hunger Index (GHI) has ranked India a lowly 102 among the 117 countries it has mapped.

Source: https://vajiramias.com/current-affairs/india-innovation-index- 2019/5da92bb41d5def5b88b44f55/ https://vajiramias.com/current-affairs/global-hunger-index/5da7dc011d5def4029dbb6d8/ https://vajiramias.com/current-affairs/school-education-quality-index- seqi/5d92e0c71d5def640e0c1551/

Q3. With reference to the Palaeochannel, consider the following statements:

1. A palaeochannel is a remnant of an inactive river or stream channel that has been filled or buried by younger sediment.

2. The Union Water Ministry has recently excavated an old, dried-up river in Prayagraj that linked the Ganga and rivers.

Which of the statements given above is/are correct? a. 1 only b. 2 only c. Both 1 and 2 d. Neither 1 nor 2

Answer: c

Explanation:

Statement 1 is correct:

A palaeochannel is a remnant of an inactive river or stream channel that has been filled or buried by younger sediment. A palaeochannel is distinct from the overbank deposits of currently-active river channels, including ephemeral water courses that do not regularly flow. Paleochannels typically act as pathways for groundwater movement and provide a potential source of groundwater. Their presence can be helpful in identifying areas suitable for recharge.

Statement 2 is correct:

The Union Water Ministry has excavated an old, dried-up river in Prayagraj (formerly Allahabad) that linked the Ganga and Yamuna rivers. The aim is to develop it as a potential groundwater recharge source. This “ancient buried river” is around 4 km wide, 45 km long and consisted of a 15-metre-thick VAJIRAM & RAVI layer buried under soil. The newly discovered river was a “buried paleochannel that joins the Yamuna river at Durgapur village, about 26 km south of the current Ganga-Yamuna confluence at Prayagraj. These paleochannels reveal the course of rivers that have ceased to exist. Evidence from palaeochannels suggested that the mythological River did indeed exist. The discovery was made in 2018 by a team of scientists from the CSIR-NGRI (National Geophysical Research Institute) and the Central Groundwater Board.

Source: https://vajiramias.com/current-affairs/paleochannel/5d92e0561d5def640e0c1531/

Q4. With reference to the Doing Business Report (DBR, 2020), consider the following statements:

1. It was released by (WEF).

2. It is not related with Ease of Doing Business Ranking.

Which of the statements given above is/are correct? a. 1 only b. 2 only c. Both 1 and 2 d. Neither 1 nor 2

Answer: d

Explanation:

The World Bank released its latest Doing Business Report (DBR, 2020). Doing Business 2020 measures regulations across 190 economies. Doing Business covers 12 areas of business regulation. Ten of these areas—starting a business, dealing with construction permits, getting electricity, registering property, getting credit, protecting minority investors, paying taxes, trading across borders, enforcing contracts, and resolving insolvency—are included in the ease of doing business score and ease of doing business ranking. Doing Business also measures regulation on employing workers and contracting with the government, which are not included in the ease of doing business score and ranking. Hence both statements 1 and 2 are incorrect.

By documenting changes in regulation in 12 areas of business activity in 190 economies, Doing Business analyzes regulation that encourages efficiency and supports freedom to do business. The data collected by Doing Business address three questions about government. First, when do governments change regulation with a view to developing their private sector? Second, what are the characteristics of reformist governments? Third, what are the effects of regulatory change on different aspects of economic or investment activity?

India has recorded a jump of 14 positions against its rank of 77 in 2019 to be placed now at 63rd rank among 190 countries. . As a result of continued efforts by the Government, India has improved its rank by 79 positions in last five years [2014-19]. India continues to maintain its first position among South Asian countries. VAJIRAM & RAVI

Source: https://vajiramias.com/current-affairs/doing-business-report- 2020/5db259321d5def7ebd092bab/

Q5. With reference to ‘The Gandhian Challenge’, consider the following statements:

1. It was launched by Ministry of Human Resource Development.

2. It provides a platform for every child across India to ideate innovative solutions for a sustainable India of their dreams, using Gandhi’s principles.

Which of the statements given above is/are correct? a. 1 only b. 2 only c. Both 1 and 2 d. Neither 1 nor 2

Answer: b

Explanation:

Statement 1 is incorrect.

On the 150th birth Anniversary of Mahatma Gandhi, Atal innovation mission (AIM), NITI Aayog’s Atal Tinkering Labs (ATL) and UNICEF India, including Generation Unlimited, have launched ‘The Gandhian Challenge’.

Statement 2 is correct.

This innovation challenge provides a platform for every child across India to ideate innovative solutions for a sustainable India of their dreams, using Gandhi’s principles. The problem statement for the Gandhian Challenge is: “Share your innovative solutions/ideas to create a futuristic and sustainable world of your dreams, following Gandhi's principles”.

Value Addition

Generation Unlimited is a new UNICEF-led global partnership that aims to ensure that every young person age 10-24 is in some form of school, learning, training, self-employment, or age-appropriate employment by 2030. It aims to co-create and scale up proven solutions related to secondary age- education, skills for learning, employability and decent work, and empowerment, with a focus on girls.

Source: https://vajiramias.com/current-affairs/the-gandhian-challenge/5d94310b1d5def0ab2c6a049/

Q6. Mediterranean Sea borders which of the following countries?

1. Israel

2. Turkey

3. Lebanon VAJIRAM & RAVI

4. Russia

Select the correct answer using the code given below: a. 1 and 4 only b. 1 and 2 only c. 2 and 3 only d. 1, 2 and 3 only

Answer: d

Explanation:

The countries surrounding the Mediterranean in clockwise order are Spain, France, Monaco, Italy, Slovenia, Croatia, Bosnia and Herzegovina, Montenegro, Albania, Greece, Turkey, Syria, Lebanon, Israel, Egypt, Libya, Tunisia, Algeria, and Morocco; Malta and Cyprus are island countries in the sea. Hence, option (d) is the correct answer.

Source: https://vajiramias.com/current-affairs/lebanon/5db3dffa1d5def24da306c72/

Q7. With reference to the National Mission (NMM), consider the following statements:

1. Ministry of Science and Technology has launched 'National Monsoon Mission'. VAJIRAM & RAVI

2. It objective is to improve the prediction of temperature, rainfall and extreme weather events on short to medium range time scale.

Which of the statements given above is/are correct? a. 1 only b. 2 only c. Both 1 and 2 d. Neither 1 nor 2

Answer: b

Explanation:

Statement 1 is incorrect:

Ministry of Earth Sciences (MoES), Government of India has launched 'National Monsoon Mission' (NMM). MoES has bestowed the responsibility of execution and coordination of this mission to the Indian Institute of Tropical Meteorology (IITM), Pune.

Statement 2 is correct:

The objective of the mission is to build an ocean atmospheric model for –improved prediction of monsoon rainfall on extended range to seasonal time scale (16 days to one season) and improved prediction of temperature, rainfall and extreme weather events on short to medium range time scale (up to 15 days).

Source: https://vajiramias.com/current-affairs/national-monsoon- mission/5d956ce71d5def0ab811082d/

Q8. With reference to Exchange-Traded Fund (ETF), consider the following statements:

1. Exchange Traded Funds are essentially Index Funds that are listed and traded on exchanges like stocks.

2. Bharat-22 ETF is managed by ICICI Prudential Asset Management Company.

Which of the statements given above is/are correct? a. 1 only b. 2 only c. Both 1 and 2 d. Neither 1 nor 2

Answer: c

Explanation:

Statement 1 is correct: VAJIRAM & RAVI

Exchange Traded Funds are essentially Index Funds that are listed and traded on exchanges like stocks. In an ETF, one can buy and sell units at a prevailing market price on a real-time basis during market hours. Index ETFs are created by institutional investors swapping shares in an index basket, for units in the fund.

Statement 2 is correct:

Bharat 22 ETF invests in the 22 companies that comprise the S&P BSE Bharat 22 index—19 companies are in the public sector and three in the private sector. An open-ended ETF, the fund allows investors to invest in a basket of government-owned and private sector entities. Fresh tranche of Bharat 22 ETF has a base size of ₹2,000 crore with the government retaining the right to keep an unspecified additional amount in case of oversubscription. The Bharat 22 ETF, is managed by ICICI Prudential Asset Management Company.

Source: https://vajiramias.com/current-affairs/further-fund-offer-2-ffo-2-of-bharat-22- etf/5d96b55d1d5def0ab6f263c5/

Q9. Which of the following glacier's melting waters are the main source of the Nubra River in the region of Ladakh? a. Nanda Devi Glacier b. c. Gangotri Glacier d. Rongbuk Glacier

Answer: b

Explanation:

The Siachen Glacier is a glacier located in the eastern Karakoram range in the Himalayas, just northeast of the point NJ9842 where the Line of Control between India and Pakistan ends. The glacier lies between the Saltoro Ridge immediately to the west and the main Karakoram range to the east. The Saltoro Ridge originates in the north from the Sia Kangri peak on the China border in the Karakoram range. The entire Siachen Glacier, with all major passes, is currently under the administration of India since 1984. Pakistan controls the region west of Saltoro Ridge, far away from the glacier. The glacier's melting waters are the main source of the Nubra River in the Indian region of Ladakh, which drains into the Shyok River. The Shyok in turn joins the 3000 kilometre-long which flows through Pakistan. Thus, the glacier is a major source of the Indus. At 76 km long, it is the longest glacier in the Karakoram and second-longest in the world's non-polar areas. Hence, option (b) is the correct answer.

Source: https://vajiramias.com/current-affairs/siachen-glacier/5db3db9a1d5def24d9351901/

Q10. What is “Einstein Challenge", recently seen in news? a. It aims to propagate the ideas of Mahatma Gandhi through innovations. b. It aims to propagate the idea of Albert Einstein’s theory of special relativity. VAJIRAM & RAVI c. It aims to propagate the issues of climate change, loss of biodiversity, over-use of natural resources and environmental and health issues. d. None of the above.

Answer: a

Explanation:

On the occasion of the Mahatma’s 150th birth anniversary, Prime Minister Narendra Modi has launched an “Einstein Challenge" to propagate the ideas of Mahatma Gandhi through innovations. He recalled Albert Einstein’s famous words on Gandhi: “Generations to come will scarce believe that such a one as this ever in flesh and blood walked upon this earth”. PM called upon thinkers, entrepreneurs and tech leaders to take forward Gandhi’s idea of innovation for future generations to remember. He said that from uniting those who believe in humanity to furthering sustainable development and ensuring economic self-reliance, Gandhi offers solutions to every problem. Hence, option (a) is the correct answer.

Source: https://vajiramias.com/current-affairs/einstein-challenge/5d96b3e61d5def0ab8114e57/

Q11. Which one of the following is the objective of ‘Mo Sarkar’ programme that was in news recently? a. It is financial inclusion program of the state of West . b. It aims to provide government service with dignity to people in . c. It is a central government pension scheme primarily targeted at the unorganised sector. d. None of the above.

Answer: b

Explanation:

‘Mo Sarkar’ programme is an initiative of Odisha government. The objective of the ‘Mo Sarkar’ programme is to provide service with dignity to people who are coming to government offices for different purposes. The phone numbers of people who are coming to government offices will be collected randomly with the purpose to improve the governance system by collecting feedback on behaviour and professionalism of government officers. The Chief Minister, Departmental Minister, Director General of Police (in case of police stations) and Departmental Minister, Secretary and Director (in case of hospitals) will call on random numbers to collect feedback. The employees will be ranked as good or bad on the basis of the feedback and those with good rank will get out-of-turn promotion and action will be taken against employees with bad rank. Hence, option (b) is the correct answer.

Source: https://vajiramias.com/current-affairs/mo-sarkar/5d96b4dc1d5def0ab3369841/

Q12. With reference to the Youth Co:Lab, consider the following statements:

1. It was launched by UNESCO.

2. It aims to accelerate youth-led social entrepreneurship and innovation in India. VAJIRAM & RAVI

Which of the statements given above is/are correct? a. 1 only b. 2 only c. Both 1 and 2 d. Neither 1 nor 2

Answer: b

Explanation:

Atal Innovation Mission (AIM), NITI Aayog and United Nations Development Programme (UNDP) India launched Youth Co:Lab to accelerate youth-led social entrepreneurship and innovation in India. Co- created in 2017 by UNDP and the Citi Foundation, and operational in 25 countries across the Asia Pacific region, the Youth Co:Lab initiative aims to create an enabling ecosystem to promote youth leadership, innovation, and social entrepreneurship. Through Youth Co:Lab, young entrepreneurs and innovators will get a chance to connect with governments, mentors, incubators and investors, who will help equip them with entrepreneurial skills. The first phase of Youth Co:Lab will focus on six SDGs: SDG 5 (Gender Equality), SDG 6 (Clean Water and Sanitation), SDG 7 (Affordable and Clean Energy), SDG 8 (Decent Work and Economic Growth), SDG 12 (Sustainable Consumption and Production) and SDG 13 (Climate Action). The initiative will also convene a series of youth dialogues across several cities such as New Delhi, Hyderabad, and to promote entrepreneurship across India. Hence only statement 2 is correct.

Source: https://vajiramias.com/current-affairs/youth-colab/5d97fb841d5def0ab2c7705d/

Q13. With reference to the new strategic disinvestment policy, consider the following statements:

1. Under the new policy, the Department of Investment and Public Asset Management (DIPAM) under the Ministry of Finance has been made the nodal department for the strategic stake sale.

2. Ministry of Heavy Industries and Public Enterprises is designated to identify PSUs for strategic disinvestment.

Which of the statements given above is/are correct? a. 1 only b. 2 only c. Both 1 and 2 d. Neither 1 nor 2

Answer: a

Explanation:

The Union Cabinet has approved a new process of strategic disinvestment with a view to expediting privatization of select PSUs. Under the new policy, the Department of Investment and Public Asset VAJIRAM & RAVI

Management (DIPAM) under the Ministry of Finance has been made the nodal department for the strategic stake sale. Hence statement 1 is correct.

DIPAM and NITI Aayog will now jointly identify PSUs for strategic disinvestment. The decision has been taken with a view to streamlining and speeding up the process. Union Finance Minister during the presentation of the Budget had announced about increasing the divestment target from 90 thousand crore rupees to 1.05 lakh crore rupees for the current fiscal year, focusing on consolidation of public sector undertakings and strategic disinvestment. Hence statement 2 is incorrect.

Source: https://vajiramias.com/current-affairs/new-strategic-disinvestment- policy/5d97fc8c1d5def0ab336de6b/

Q14. Which one of the following statements best describes the objective of “Acceptance Development Fund (ADF)”, recently in news? a. It was set up by Department for Promotion of Industry and Internal Trade (DPIIT) to boost startups in India. b. It is created to increase acceptance of debit and credit cards in tier III and tier VI cities. c. It is a venture capital fund to promote micro, small and medium enterprises (MSMEs). d. None of the above

Answer: b

Explanation:

RBI has said that the framework for Acceptance Development Fund (ADF) is to be set up to increase acceptance of debit and credit cards in tier III and VI cities. With the rapid growth in the issuance of cards in the country, there is a need to ensure growth of acceptance infrastructure across the country, especially in Tier III to Tier VI centres. In order to increase digitisation in these areas, it has been decided to create an 'Acceptance Development Fund' (ADF) in consultation with the stakeholders – the card issuers (banks) and card payment networks (Visa, MasterCard and RuPay), the government and RBI. Currently, India has 42 lakh PoS machines, but with the help of this fund, the penetration into tier-III and tier IV centres is expected to rise. The government's contribution to the ADF would come from NABARD's Financial Inclusion Fund (FIF) and RBI's contribution from the Depositors' Education and Awareness (DEA) Fund. This was indicated in the Payment System Vision Document 2021 of RBI and also recommended by the Committee on Deepening of Digital Payments (Chaired by Nandan Nilekani). The framework will be operationalised by December 2019. Hence, option (b) is the correct answer.

Source: https://vajiramias.com/current-affairs/acceptance-development-fund- adf/5d99496b1d5def0ab6f2e849/

Q15. Which of the following is/are named as “Green crackers”, recently seen in news?

1. Safe Water Releaser (SWAS)

2. Safe Thermite Cracker (STAR)

3. Safe Minimal Aluminium (SAFAL) VAJIRAM & RAVI

Select the correct answer using the code given below: a. 1 only b. 1 and 2 only c. 2 and 3 only d. 1, 2 and 3

Answer: d

Explanation:

“Green crackers” are so named because they “do not contain harmful chemicals” that would cause air pollution. Components in firecrackers are replaced with others that are “less dangerous” and “less harmful” to the atmosphere. Scientists have given these crackers names: Safe Water Releaser (SWAS), Safe Thermite Cracker (STAR) and Safe Minimal Aluminium (SAFAL).

Council of Scientific and Industrial Research (CSIR) labs have been successful in developing various environment-friendly fireworks such as sound emitting crackers, flowerpots, pencils, chakkar and sparklers. The emissions testing facilities for the new fire crackers have been set up at CSIR-NEERI. Further, a Raw Materials Compositional Analysis (RACE) facility has been launched in Sivakasi to facilitate manufacturers for testing their raw materials and chemicals. Hence, option (d) is the correct answer.

Source: https://vajiramias.com/current-affairs/green-crackers/5d994a601d5def0ab6f2e872/

Q16. With reference to the geotail, consider the following statements:

1 The region exists as a result of the interactions between the Moon and Earth.

2. Once every 29 days, the Moon traverses the geotail for about six days.

Which of the statements given above is/are correct? a. 1 only b. 2 only c. Both 1 and 2 d. Neither 1 nor 2

Answer: b

Explanation:

The geotail is a region in space that allows the best observations. The region exists as a result of the interactions between the Sun and Earth. It provides information about the way the magnetic envelope surrounding Earth, called the magnetosphere, responds to incoming material and energy from the Sun. Once every 29 days, the Moon traverses the geotail for about six days. When Chandrayaan-2, which is orbiting the Moon, crosses the geotail, its instruments can study the properties of the geotail. The Sun VAJIRAM & RAVI emits the solar wind, which is a continuous stream of charged particles. These particles are embedded in the extended magnetic field of the Sun. Since the Earth has a magnetic field, it obstructs the solar wind plasma. This interaction results in the formation of a magnetic envelope around Earth. On the Earth side facing the Sun, the envelope is compressed into a region that is approximately three to four times the Earth radius. On the opposite side, the envelope is stretched into a long tail, which extends beyond the orbit of the Moon. It is this tail that is called the geotail. Hence only statement 2 is correct.

Source: https://vajiramias.com/current-affairs/geotail/5d9aa5171d5def18d9f0bc47/

Q17. With reference to Planet Nine, consider the following statements:

1. It is a recently found dwarf planet orbiting Mars in the Solar System.

2. Its gravitational effects of Planet Nine are not observed in the Solar system.

Which of the statements given above is/are correct? a. 1 only b. 2 only c. Both 1 and 2 VAJIRAM & RAVI d. Neither 1 nor 2

Answer: d

Explanation:

Planet Nine is a hypothetical planet in the outer region of the Solar System. As of 2019-20, no observation of Planet Nine had been announced. Its gravitational effects could explain the unusual clustering of orbits for a group of extreme trans-Neptunian objects (eTNOs), bodies beyond Neptune that orbit the Sun at distances averaging more than 250 times that of the Earth. Over the years, scientists have sought to explain several puzzling aspects of the Solar System by attributing these to the influence of Planet Nine. In a 2016 paper, researchers made out a case for Planet Nine’s existence by arguing that it could be responsible for the peculiar alignment of icy objects on the outskirts of the Solar System.

So, both the statements incorrect.

Source: https://vajiramias.com/current-affairs/planet-nine/5d9aa3a51d5def18d791c2e0/

Q18. With reference to the MOSAiC mission, consider the following statements:

1. Its objective is to study the impact of climate change on the Arctic and rest of the world.

2. It the largest ever Arctic expedition in history.

Which of the statements given above is/are correct? a. 1 only b. 2 only c. Both 1 and 2 d. Neither 1 nor 2

Answer: c

Explanation:

The MOSAiC (Multidisciplinary drifting Observatory for the Study of Arctic Climate) mission stands for Multidisciplinary drifting Observatory for the Study of Arctic Climate. Mission objective is to study the impact of climate change on the Arctic and how it could affect the rest of the world.

For the first time a modern research icebreaker will operate in the direct vicinity of the North Pole year round, including the nearly half year long polar night during winter. In terms of the logistical challenges involved, the total number of participants, the number of participating countries, and the available budget, MOSAiC represents the largest Arctic expedition in history. It will be the first to conduct a study of this scale at the North Pole for an entire year. Previous studies have been of shorter periods as the thicker sea ice sheets prevent access in winter.

Hence both statements are correct.

Source: https://vajiramias.com/current-affairs/mosaic-mission/5d9aa4011d5def18d9f0bc23/ VAJIRAM & RAVI

Q19. With reference to the Bharat Stage Six (BS-VI) vehicular emission norms, consider the following statements:

1. The standards and the timeline for implementation are set by the Central Pollution Control Board.

2. The amount of Sulfur in the fuel is reduced to 50 ppm in BS VI fuel for both gasoline and diesel.

Which of the statements given above is/are correct? a. 1 only b. 2 only c. Both 1 and 2 d. Neither 1 nor 2

Answer: a

Explanation:

Statement 1 is correct:

Bharat Stage norms are the automotive emission norms which the automotive manufacturers have to comply to sell their vehicles in India. These norms are applicable to all two wheelers, three wheelers, four wheelers and construction equipment vehicles. Its objective is to regulate the output of air pollutants from internal combustion engine equipment. The higher the fuel standard a vehicle complies with, the less polluting it is. These are emission standards instituted by the Government of India based on European regulations. The standards and the timeline for implementation are set by the Central Pollution Control Board under the Ministry of Environment & Forests and Climate Change. The Government of India has decided to leapfrog from the exiting BS – IV norms to the BS- VI, thereby skipping the BS – V norms, and to implement the BS – VI norms with effect from 1st April 2020.

Statement 2 is incorrect:

The main difference is in the amount of sulfur in the fuel which is reduced from 50 ppm in BS IV fuel to 10 ppm in BS VI fuel for both gasoline and diesel. BS VI norms will address one of the inherent flaws in the European emission standards which permits diesel cars to emit more particulate matter and nitrogen oxide. NOx emissions from diesel cars are expected to come down by nearly 70% and, from cars with petrol engines, by 25%. BS-VI sets limits set on Particle Number (PN) for engines, a reference to direct injection engines that emit more particulates but are more efficient and release less carbon dioxide.

Source: https://vajiramias.com/current-affairs/bs-vi-norms/5d9c126d1d5def22b28dbd79/

Q20. With reference to the Indian Technical and Economic Cooperation (ITEC) Programme, consider the following statements:

1. It was launched by central government to help development of the backward states.

2. It is fully funded by the Government of India. VAJIRAM & RAVI

Which of the statements given above is/are correct? a. 1 only b. 2 only c. Both 1 and 2 d. Neither 1 nor 2

Answer: b

Explanation:

Indian Technical and Economic Cooperation Programme was launched in 1964 by the Ministry of External Affairs. The ITEC Programme is fully funded by the Government of India. The Programme is essentially bilateral in nature. However, in recent years, ITEC resources have also been used for cooperation programmes conceived in regional and inter-regional context such as Economic Commission for Africa, Commonwealth Secretariat, UNIDO and Group of 77.It has the following components: Training (civilian and defence) in India of nominees from ITEC partner countries; Projects and project related activities such as feasibility studies and consultancy services; Deputation of Indian experts abroad; Study Tours; Gifts/Donations of equipment at the request of ITEC partner countries; and Aid for Disaster Relief. Division of Development Partnership Administration (DPA) in the Ministry of External Affairs is the nodal division for handling all capacity building programmes. Hence only statement 2 is correct.

Source: https://vajiramias.com/current-affairs/indian-technical-and-economic-cooperation-itec- programme/5d9c14e71d5def22b7590a7b/

Q21. What is ‘INS BAAZ’, recently seen in news? a. The southernmost air station of the b. A conventional diesel-electric submarine of the . c. Third ship of the Visakhapatnam-class stealth guided-missile destroyers of the Indian Navy. d. None of the above

Answer: a

Explanation:

The Indian Naval Ship (INS) ‘Baaz’ was commissioned in July 2012. It is the southernmost air station of the Indian Armed Forces. INS Baaz is located at Campbell Bay on the , the southernmost and largest island of the , in the UT of Andaman and Nicobar Islands. This island is also the location of the Indira Point. The primary functions of the INS Baaz include helping build Maritime Domain Awareness by providing information via airborne surveillance using aircraft and Unmanned Aerial Vehicles (UAVs). The location has been described as India’s “window into East and Southeast Asia”, and is in close vicinity of the Six Degree Channel, one of the Indian Ocean’s busiest shipping lines, carrying strategic cargo to East Asian countries. It is also close to the . It VAJIRAM & RAVI is a part of the Andaman and Nicobar Command, the only tri-services formation of the Indian Armed Forces that was started in 2001. Hence, option (a) is the correct answer.

Source: https://vajiramias.com/current-affairs/ins-baaz/5db7ae201d5def24da30b30c/

Q22. With reference to the Graded Response Action Plan (GRAP), consider the following statements:

1. The plan requires action and coordination among all central and state agencies in India.

2. It works only as an emergency measure.

Which of the statements given above is/are correct? a. 1 only b. 2 only c. Both 1 and 2 d. Neither 1 nor 2

Answer: b

Explanation:

Statement 1 is incorrect:

Graded Response Action Plan (GRAP) was approved by the Supreme Court in 2016, GRAP was notified in 2017 by the Centre and draws its authority from this notification. The plan requires action and coordination among 13 different agencies in Delhi, , Haryana and (NCR areas). At the head of the table is the EPCA, mandated by the Supreme Court.

Statement 2 is correct:

GRAP works only as an emergency measure. As such, the plan does not include action by various state governments to be taken throughout the year to tackle industrial, vehicular and combustion emissions. When the air quality shifts from poor to very poor, the measures listed under it have to be followed since the plan is incremental in nature. If air quality reaches the severe+ stage, GRAP talks about shutting down schools and implementing the odd-even road-space rationing scheme. The EPCA, headed by retired IAS officer Bhure Lal and including members from the Centre for Science and Environment, was constituted in 1998 by the Supreme Court. The initial mandate of the body was to ensure the shift of Delhi’s bus and auto fleet to CNG — a mammoth task that was among the most crucial ones in cleaning Delhi’s air in the late 2000s. The body continues to monitor pollution and assists the Supreme Court in several pollution-related matters.

Source: https://vajiramias.com/current-affairs/graded-response-action-plan- grap/5d9d747c1d5def38e81a7c23/

Q23. With reference to the United Nations (UN) Universal Postal Union (UPU), consider the following statements:

1. It is a specialized agency of the United Nations (UN) with its Headquarters in Switzerland. VAJIRAM & RAVI

2. It was established by the Treaty of Bern in 1874.

Which of the statements given above is/are correct? a. 1 only b. 2 only c. Both 1 and 2 d. Neither 1 nor 2

Answer: c

Explanation:

Statement 1 is correct:

The Universal Postal Union (UPU) is a specialized agency of the United Nations (UN). It coordinates postal policies among member nations, in addition to the worldwide postal system. It frames rules for international mail exchange, and fixes rates for international postal services. It regulates 6.40 lakh postal outlets worldwide. The UPU has 192 member-countries. Headquarters is at Bern, Switzerland.

Statement 2 is correct:

It was established by the Treaty of Bern in 1874. India joined the UPU on July 1, 1876. The UPU has four units: the Congress, the Council of Administration (CA), the Postal Operations Council (POC) and the International Bureau (IB). Under UPU rules, when a country decides to suspend exchange with a country, it must notify the operator of the other country (in India’s case, India Post) and, if possible, the duration for which services are being stopped. The UPU’s International Bureau too has to be notified.

Source: https://vajiramias.com/current-affairs/the-united-nations-universal-postal-union- upu/5db7b4191d5def24da30b358/

Q24. Which of the following diseases are classified as Neglected Tropical Diseases by World Health Organization (WHO)?

1. Leprosy

2. Lymphatic Filariasis

3. Dengue

Select the correct answer using the code given below: a. 3 only b. 1 and 2 only c. 2 and 3 only d. 1, 2 and 3 VAJIRAM & RAVI

Answer: d

Explanation:

Point (1) is correct: Leprosy, also known as Hansen's disease, is a chronic infectious disease caused by Mycobacterium leprae. It is classified as Neglected Tropical Diseases (NTDs).

Point (2) is correct: Lymphatic Filariasis, commonly known as elephantiasis, is a Neglected Tropical Diseases (NTDs). According to the WHO, infection occurs when filarial parasites are transmitted to humans through mosquitoes. The infection is usually acquired in childhood, causing hidden damage to the lymphatic system.

Point (3) is correct: Dengue fever is a mosquito-borne tropical disease caused by the dengue virus. It is classified as Neglected Tropical Diseases (NTDs).

Source: https://vajiramias.com/current-affairs/dengue/5d9d71891d5def38e81a7b7e/

Q25. Which of the following region is known as the 'Third Pole of the Earth'? a. Central Siberian plateau b. Iceland and Greenland c. Hindu- Kush- Himalayan (HKH) region d. None of the above

Answer: c

Explanation:

The Hindu-Kush-Himalayan (HKH) region spans Afghanistan, , Bhutan, China, India, Kyrgyzstan, Mongolia, Myanmar, Nepal, Pakistan, Tajikistan, and Uzbekistan. The HKH region is considered the Third Pole [after the North and South Poles] and has significant implications for climate. VAJIRAM & RAVI

The Third Pole, which contains vast cryospheric zones, is also the world’s largest store of snow and ice outside the polar region, and the source of 10 major rivers, and, therefore, particularly sensitive to climate change. Earlier this month, the IMD organised a workshop to discuss ways to establish a regional climate centre that will provide forecasting services and climate analyses. It will be under the World Meteorological Organisation (WMO) and take a few years to take shape. Alongside forecasting weather over long periods, the regional centres would provide data services, training and capacity-building, research and development. Hence, option (c) is the correct answer.

Source: https://vajiramias.com/current-affairs/hindu-kush-himalayan-hkh- region/5d9d73fb1d5def38e7ffd626/

Q26. With reference to the National Health Mission (NHM), consider the following statements:

1. ASHAs are provided the cover of both Pradhan Mantri Jevan Jyoti Beema Yojna and Pradhan Mantri Suraksha Beema Yojna

2. Home Based Care for Young Child (HBYC) programme was introduced under POSHAN Abhiyaan.

Which of the statements given above is/are correct? a. 1 only b. 2 only c. Both 1 and 2 d. Neither 1 nor 2

Answer: c

Explanation:

The National Health Mission (NHM) envisages achievement of universal access to equitable, affordable & quality health care services that are accountable and responsive to people's needs. The Union Cabinet has been apprised of the Progress under National Health Mission (NHM) and Decisions of the Empowered Programme Committee and Mission Steering Group of the NHM.

Key elements:

• There has been acceleration in decline of MMR, Under Five Mortality Rate (U5MR) and the IMR since the launch of the NRHM/NHM. At the current rate of decline, India should able to reach its SDG target (MMR-70, U5MR-25) much before the due year i.e. 2030. • Revised National Tuberculosis Control Programme (RNTCP) has been strengthened. Due to this, there is 16% jump in identification of new cases in one year. Newer drug regimen of Bedaquiline and Delaminide and nutrition support to all the TB patients for the duration of the treatment has been rolled throughout the country. • The routine and recurring incentives of ASHAs got increased from 1000 per month to 2000 per month. ASHAs and ASHA Facilitators were provided the cover of Pradhan Mantri Jevan Jyoti BeemaYojna and Pradhan Mantri Suraksha Beema Yojna. VAJIRAM & RAVI

• During 2018-19, Rotavirus vaccine (RVV) was introduced in additional two States. Till today, all the States/UTs are covered with RVV. • During 2018-19, Pneumococcal Conjugated Vaccine (PCV) was expanded to MP, Haryana and the remaining districts of , Rajasthan and Uttar Pradesh. • Anaemia Mukt Bharat (AMB) Abhiyaan was launched under POSHAN Abhiyaan in April 2018. • Home Based Care for Young Child (HBYC) programme was introduced under POSHAN Abhiyaan. • National Viral Hepatitis Control Programme was approved for prevention, management and treatment of Hepatitis to A, B, C and E and rollout was initiated. Hence both statements are correct.

Source: https://vajiramias.com/current-affairs/national-health-mission- nhm/5d9eb4171d5def4825f01a56/

Q27. Which of the following are the strategic priorities under World Health Organization (WHO) India Country Cooperation Strategy 2019–2023?

1. Accelerating progress on universal health coverage

2. Protecting the population against health emergencies

3. Enhancing India's global leadership in health

Select the correct answer using the code given below: a. 1 and 3 only b. 1 and 2 only c. 2 and 3 only d. 1, 2 and 3

Answer: d

Explanation:

Union Health Ministry recently launched ‘The WHO India Country Cooperation Strategy 2019–2023: A Time of Transition’. The Country Cooperation Strategy (CCS) provides a strategic roadmap for WHO to work with the Government of India towards achieving its health sector goals. The CCS outlines how WHO can support the Ministry of Health and other allied Ministries to drive impact at the country level. WHO's support to the government will fall under four strategic priorities:

1. accelerating progress on universal health coverage, 2. promoting health and wellness, 3. protecting the population against health emergencies, and 4. enhancing India's global leadership in health. The India CCS is one of the first that fully aligns itself with the newly adopted WHO 13th General Programme of Work and its 'triple billion' targets, the Sustainable Development Goals (SDGs) and WHO South-East Asia Region’s eight Flagship Priorities. It captures the work of the United Nations Sustainable Development Framework for 2018–2022. Hence, option (d) is the correct answer. VAJIRAM & RAVI

Source: https://vajiramias.com/current-affairs/who-india-country-cooperation-strategy- 20192023/5d9ebb821d5def4822b26105/

Q28. With reference to the Gagan Enabled Mariner’s Instrument for Navigation and Information (GEMINI) device, consider the following statements:

1. It has been developed by Indian National Centre for Ocean Information Services (INCOIS), and Airports Authority of India (AAI).

2. The device is to help in providing information related to disaster warnings when fishermen move away from the coast beyond 10 to 12 kilometres.

Which of the statements given above is/are correct? a. 1 only b. 2 only c. Both 1 and 2 d. Neither 1 nor 2

Answer: c

Explanation:

Union Earth Sciences Ministry recently launched the Gagan Enabled Mariner’s Instrument for Navigation and Information (GEMINI) device. GEMINI device will disseminate seamless and effective dissemination of emergency information and communication on disaster warnings, Potential Fishing Zones (PFZ) and Ocean States Forecasts (OSF) to fishermen. The device will help to provide information related to disaster warnings when fishermen move away from the coast beyond 10 to 12 kilometres. The GEMINI device receives and transfers the data received from GAGAN satellite/s to a mobile through Bluetooth communication. A mobile application developed by INCOIS decodes and displays the information in nine regional languages. It has been developed by Indian National Centre for Ocean Information Services (INCOIS), and Airports Authority of India (AAI). It is electronically designed and manufactured by a private industry M/S Acord, Bangalore under Make in India Program. Hence both statements are correct.

Source: https://vajiramias.com/current-affairs/gagan-enabled-mariners-instrument-for-navigation- and-information-gemini/5d9eba0a1d5def4827b6ce0f/

Q29. Which of the following Planet has the most known moons in the solar system? a. Saturn b. Neptune c. Jupiter d. Venus

Answer: a VAJIRAM & RAVI

Explanation:

The International Astronomical Union’s Minor Planet Center confirmed 20 new moons orbiting Saturn, making it the planet with the most moons in our Solar System, at 82. The 20 had been discovered by Scott S Sheppard of the Carnegie Institution for Science. Until their confirmation, the planet with the most moons was Jupiter, at 79. Each of the newly discovered objects in orbit around Saturn is about 5km (three miles) in diameter; 17 of them orbit the planet "backwards". This is known as a retrograde direction. The other three moons orbit in a prograde direction - the same direction as Saturn rotates. A count of the moons listed on the NASA website shows that our Solar System’s planets together have 205 confirmed moons now. Saturn and Jupiter, with 161 between them, account for nearly 80% of these. Another 20% are orbiting Uranus (27) and Neptune (14). Of the remaining three moons, one is Earth’s own while the other two are with Mars. Hence, option (a) is the correct answer.

Value Addition:

Mercury is so close to the Sun and its gravity that it wouldn’t be able to hold on to its own moon. Any moon would most likely crash into Mercury or maybe go into orbit around the Sun and eventually get pulled into it. It is not yet clear, however, why Venus does not have a moon.

Source: https://vajiramias.com/current-affairs/saturns-moon/5d9ebb151d5def4824d4aeb0/

Q30. With reference to the Global Competitiveness Index 2019, consider the following statements:

1. It is released annually by the World Bank.

2. Within South Asia, Sri Lanka is the best performer.

Which of the statements given above is/are correct? a. 1 only b. 2 only c. Both 1 and 2 d. Neither 1 nor 2

Answer: d

Explanation:

Statement 1 is incorrect:

The latest edition of the Global Competitiveness Report ranks India at 68th position among 141 countries – that’s 10 ranks below its 2018 position in the same index. The annual Global Competitiveness Index is compiled by the Geneva-based World Economic Forum (WEF). It was first launched in 1979. This is the fourth version of the global competitiveness index – hence referred to as GCI 4.0 – and it was introduced in 2018. GCI maps the factors that determine the Total Factor Productivity (TFP) in a country. The GCI 4.0 maps the competitiveness landscape of 141 economies through 103 indicators organised into 12 pillars which are further divided into 4 broad categories namely (1) Enabling Environment, (2) Human Capital, (3) Markets and (4) Innovation Ecosystem. A VAJIRAM & RAVI country’s performance on the GCI results is reported as a ‘progress score’ on a 0-to-100 scale, where 100 represents the ‘frontier’, an ideal state where an issue ceases to be a constraint to productivity growth”.

Statement 2 is incorrect:

Singapore has become the world’s most competitive economy in 2019, pushing the US to the second place. Hong Kong SAR is ranked 3rd, Netherlands is 4th and Switzerland is ranked 5th. India’s 2019 overall score (61.4) fell by merely 0.7 when compared to its 2018 score. But this slippage was enough for it to slide down 10 ranks in the list to 68th position. India is among the worst-performing BRICS nations along with Brazil (ranked even lower than India at 71 this year). India trails China (28th, 73.9) by 40 places and 14 points. Within South Asia, it is the best performer and is followed by Sri Lanka (84th), Bangladesh (105th), Nepal (108th) and Pakistan (110th). India is ranked second globally for shareholder governance, third in terms of the market size and third in terms of renewable energy regulation.

Source: https://vajiramias.com/current-affairs/global-competitiveness- index/5da011e01d5def48268e89bd/

Q31. In the context of which of the following institutions the terms ‘Doha Declaration and Special and Differential (S&D) treatment’ is often found in the news? a. WTO affairs b. SAARC affairs c. UNFCCC affairs d. World Bank affairs

Answer: a

Explanation:

The WTO agreements contain special provisions which give developing countries special rights and allow other members to treat them more favourably. These are “special and differential treatment provisions” (abbreviated as S&D or SDT).

The special provisions include: o longer time periods for implementing agreements and commitments o measures to increase trading opportunities for these countries o provisions requiring all WTO members to safeguard the trade interests of developing countries o support to help developing countries build the infrastructure to undertake WTO work, handle disputes, and implement technical standard o provisions related to least-developed country (LDC) members Currently, any WTO member can designate itself as a developing country and avail these benefits. The US had submitted its suggestions to the WTO which states that self-declaration puts the WTO on a path to failed negotiations and it is also a path to institutional irrelevance. In the Doha Declaration, ministers VAJIRAM & RAVI agreed that all special and differential treatment provisions should be reviewed, in order to strengthen them and make them more precise, effective and operational. Hence, option (a) is the correct answer.

Source: https://vajiramias.com/current-affairs/special-and-differential- treatment/5d9ffa5c1d5def48268e859a/

Q32. With reference to the C40 Cities Climate Leadership Group (C40), consider the following statements:

1. It is a network of the world’s megacities committed to addressing climate change.

2. All Tier-1 cities from India are part of the C40.

Which of the statements given above is/are correct? a. 1 only b. 2 only c. Both 1 and 2 d. Neither 1 nor 2

Answer: a

Explanation:

C40 is a network of the world’s megacities committed to addressing climate change. The C40 group was started in 2005 by the then Mayor of London, Ken Livingstone. It has 96 members at present, representing over 70 crore people, and one-quarter of the global economy. The cities from India that are part of the C40 are Delhi NCR, Bengaluru, , and Kolkata. The C40 World Mayors’ Summit is a three-day conference where city leaders from around the world share ideas on green urban development. The summit is being held in Copenhagen (Denmark’s capital) and attendees include mayors representing over 90 cities from around the world. Over the past decade, C40 has convened six Mayors Summits, hosted by London (2005), New York (2007), Seoul (2009), Sao Paulo (2011), Johannesburg (2014) and Mexico City (2016). Hence only statement 1 is correct.

Source: https://vajiramias.com/current-affairs/c40-cities-climate-leadership-group- c40/5da010451d5def4827b70d96/

Q33. With reference to Election manifestos, consider the following statements:

1. The Model Code of Conduct (MCC) doesn’t cover the Election Manifestos under its ambit.

2. Under Representation of the People Act, 1951, political parties could be held liable for not fulfilling promises made in their election manifestos.

Which of the statements given above is/are correct? a. 1 only b. 2 only VAJIRAM & RAVI c. Both 1 and 2 d. Neither 1 nor 2

Answer: d

Explanation:

A manifesto is a published declaration of the intentions, motives, or views of the issuer, be it an individual, group, political party or government. In India, Election manifestos are not legally enforceable documents. There is no provision in law under which political parties could be held liable for not fulfilling promises made in their election manifestos. Guidelines issued by the Election Commission of India in 2013 on election manifestos in the Model Code of Conduct (MCC): The election manifesto shall not contain anything repugnant to the ideals enshrined in the Constitution. Political parties should avoid making promises which are likely to vitiate the purity of the election process or exert undue influence on the voters in exercising their franchise. It is expected that manifestos also reflect the rationale for the promises and broadly indicate the ways and means to meet the financial requirements for it.

In case of single-phase election, manifesto shall not be released during the prohibitory period, as prescribed under Section 126 of The Representation of the People Act, 1951. In case of multi-phase elections, manifesto shall not be released during the prohibitory periods of all the phases of those elections. In Section 126 of the RP Act, the ‘prohibitory period’ signifies the “period of forty-eight hours ending with the hour fixed for conclusion of poll”. Hence both statements are incorrect.

Source: https://vajiramias.com/current-affairs/election-manifestos/5da168221d5def798813a743/

Q34. "Tappattam" and "Poi kal kuthirai" are the folk dances belongs to which of the following states? a. Kerala b. c. d.

Answer: b

Explanation:

"Tappattam," and "poi kal kuthirai" are folk dances in the state of Tamil Nadu. A hand held drum instrument called Thappu is used for Thappattam dance. Thappattam is a folk dance accompanied by emotional and entertaining drum music. Poikkaal Kuthirai Aattam is a type of dance performed with a dummy horse having a gap inside so that a person can fit into it to perform the dance. This famous Tamil folk dance originated in . It is accompanied by the Kundala vadhyam or Gondal drums, native to the state of Maharashtra. Hence, option (b) is the correct answer.

Source: https://vajiramias.com/current-affairs/veshti/5da2b0701d5def79891ef2b2/

Q35. Consider the following fauna:

1. Javan Tiger VAJIRAM & RAVI

2. Siberian Tiger

3. Tasmanian Tiger

Which of the above is/are listed as Extinct on IUCN Red List of Threatened species? a. 1 only b. 1 and 3 only c. 2 and 3 only d. None of the above

Answer: b

Explanation:

Point (1) is correct: The Javan tiger was a Panthera tigris sondaica population, which lived in the Indonesian island of Java until the mid-1970s. It was hunted to extinction and its natural habitat converted for human use. It was one of the three tiger populations in the Sunda Islands.

Point (2) is not correct: The Siberian tiger is a Panthera tigris tigris population in the Russian Far East and Northeast China, and possibly North Korea. It is listed as endangered on IUCN Red list.

Point (3) is correct: The thylacine is commonly known as the Tasmanian tiger because of its striped lower back. Its IUCN status is Extinct. The last known thylacine died in captivity over 80 years ago, in Tasmania’s Hobart Zoo in 1936. It was native to Tasmania, New Guinea, and the Australian mainland. The thylacine appeared like a medium-to-large-size dog, except for its stiff tail and abdominal pouch similar to a kangaroo, and dark transverse stripes that radiated from the top of its back, reminiscent of a tiger.

Source: https://vajiramias.com/current-affairs/tasmanian-tiger/5dabc6bd1d5def1892592509/

Q36. With reference to the river dolphin, consider the following statements:

1. It is known as susu due to strange sound it makes.

2. It uses echolocation to navigate and hunt.

Which of the statements given above is/are correct? a. 1 only b. 2 only c. Both 1 and 2 d. Neither 1 nor 2

Answer: c

Explanation: VAJIRAM & RAVI

Ganges river dolphin is among the four ‘obligate’ freshwater dolphins in the world. Its presence indicates the health of the riverine ecosystem. The animal is known to make strange sounds when it breathes, earning it the sobriquet 'Susu'. Being a mammal, it has to come to the surface to breathe. It is also called a blind dolphin because it doesn’t have a crystalline eye lens and uses echolocation to navigate and hunt. The government of India declared it the National Aquatic Animal in 2009. It is listed as endangered on IUCN Red list. Hence both statements are correct.

Source: https://vajiramias.com/current-affairs/annual-ganges-river-dolphin- census/5da3d09a1d5def105221aa2b/

Q37. With reference to Index of Industrial Production (IIP), consider the following statements:

1. It is computed and published by the Central Statistical Organisation (CSO) on a monthly basis.

2. Present IIP figures are calculated considering 2005-06 as base year.

Which of the statements given above is/are correct? a. 1 only b. 2 only c. Both 1 and 2 d. Neither 1 nor 2

Answer: a

Explanation:

The Index of Industrial Production (IIP) is an index which shows the growth rates in different industry groups of the economy in a stipulated period of time. The IIP index is computed and published by the Central Statistical Organisation (CSO) on a monthly basis. Currently IIP figures are calculated considering 2011-12 as base year. Besides the new base year, different sectors have been amended based on National Industrial Classification (NIC), 2008.

Major changes in coverage of the new series of IIP:

• The un-organised sector was included in the existing series • Mining sector basket now comprises of 29 minerals as opposed to 61 minerals in the old series. • Under ‘Electricity’ sector, electricity generation from renewable energy sources has been included. There are two ways in which IIP data can be viewed.

• Broad sectors: The first is to look at sectoral performance. In this the whole industrial economy is divided into three sectors; manufacturing (with a weight of 77.63 % in the index), mining (14.37 %) and electricity (7.99 %). • Use-based sectors: The second way is to look at the way such industrial products are used, namely Basic Goods, Capital Goods and Intermediate Goods. Hence only statement 1 is correct. VAJIRAM & RAVI

Source: https://vajiramias.com/current-affairs/index-of-industrial-production- iip/5da3ea1f1d5def105809a481/

Q38. With reference to the LOTUS-HR project, consider the following statements:

1. It was launched by Ministry of Earth Sciences.

2. It aims to demonstrate a novel waste water management approach to produce clean water for reuse.

Which of the statements given above is/are correct? a. 1 only b. 2 only c. Both 1 and 2 d. Neither 1 nor 2

Answer: b

Explanation:

Statement 1 is incorrect:

India and Netherlands launched the second phase of the LOTUS-HR (Local Treatment of Urban Sewage streams for Healthy Reuseplant) as a part of joint collaboration. The LOTUS-HR project is jointly supported by Department of Biotechnology, Ministry of Science and Technology, Government of India and Netherlands Organization for Scientific Research /STW, Government of Netherlands.

Statement 2 is correct:

The project was initiated in July 2017 and aims to demonstrate a novel holistic (waste) water management approach that will produce clean water which can be reused for various purposes. In the second phase, 10,000 Litre sewage water will be treated per day.

Source: https://vajiramias.com/current-affairs/lotus-hr/5da5611b1d5def7d406c1fc3/

Q39. Which of the following pairs is/are correctly matched?

Select the correct answer using the code given below: a. 1 only VAJIRAM & RAVI b. 1 and 2 only c. 2 and 3 only d. 1, 2 and 3

Answer: a

Explanation:

Pair (1) is correctly matched: Ecuador is a country in South America, bordered by Colombia on the north, Peru on the east and south, and the Pacific Ocean on the west. Ecuador also includes the Galápagos Islands in the Pacific. The capital city is Quito. Guayaquil is a city in Ecuador. It is located on the west bank of the Guayas River, which flows into the Pacific Ocean at the Gulf of Guayaquil. Guayaquil is constantly facing tsunami and major earthquake threats due to its soil stratigraphy and location near the Gulf of Guayaquil and the south of North-Andean subduction zone.

Pair (2) is not correctly matched: Tulagi Island is a small island in Solomon Islands. Tulagi, an island about two square kilometres with a population of 1,200, is the site of a former Japanese naval base and was the scene of fierce fighting in World War II. The island of Tulagi served as a South Pacific headquarters for Britain and then Japan. During Second World War, its natural deepwater harbour across from Guadalcanal was coveted by the military. It was in news due to the "strategic cooperation agreement" on Tulagi island signed between Solomon Islands and state-owned China Sam Enterprise Group for the development of the special economic zone.

Pair (3) is not correctly matched: Marawah Island is a low-lying island off the coast of Emirate of Abu Dhabi, the United Arab Emirates. The island is a key center of archaeology. Archeologists have identified various sites dating from the Neolithic to the Islamic Period (12-13 AD). Hence, option (a) is the correct answer.

Source: https://vajiramias.com/current-affairs/guayaquil/5da5616a1d5def7d3dc14500/ https://vajiramias.com/current-affairs/tulagi-island/5da92a7e1d5def5b868d3a45/ https://vajiramias.com/current-affairs/marawah-island/5dae8c0e1d5def457c517836/

Q40. Which of the following initiative(s) is/ are associated with Climate Change?

1. Fridays for Future Movement

2. Global Solar Council (GSC)

3. Global Climate Action Summit

4. Under2 Coalition

Select the correct answer using the code given below: a. 3 only b. 2 and 3 only VAJIRAM & RAVI c. 2, 3 and 4 only d. 1, 2, 3 and 4

Answer: d

Explanation:

Point (1) is correct- Fridays for Future (FFF) is also known variously as the school strike for the climate, Youth for Climate, Climate Strike, Youth Strike for Climate. It is an international movement of school students who take time off from class to participate in demonstrations to demand action to prevent further global warming and climate change. Publicity and widespread organising began after Swedish schoolgirl Greta Thunberg staged a protest in August 2018 outside the Swedish Riksdag, holding a sign that read "School strike for the climate".

Point (2) is correct- The Global Solar Council is an international non-profit association of the national, regional and international associations in solar energy and the world‘s leading corporations. It was launched at the COP 21 climate change meeting in Paris. The GSC came into being as International Coalition of more than 30 nations, utilising maximum solar energy, decided to harness the renewable energy for the greater good.

Point (3) is correct- The Global Climate Action Summit (GCAS) 2018, in San Francisco, was the first ever global climate summit, focused exclusively on the actions and ambitions of businesses, cities, states and citizens worldwide. It bought together US and international leaders who are committed to turning the tide in the fight against climate change. A successful GCAS 2018 will now help to reassure the international community that this will not decelerate the “decarbonisation” movement. Participation of non-state actors made the Global Climate Action Summit different. This Summit and its Call to Action make an important contribution towards achieving the collective goal: to keep global temperatures to 1.5 degrees Celsius in line with the Paris Agreement.

Point (4) is correct- The Under2 Coalition is driven by a group of ambitious state and regional governments committed to keeping global temperature rises to well below 2°C. The coalition is made up of more than 220 governments who represent over 1.3 billion people and 43% of the global economy. The Climate Group is the Secretariat to the Under2 Coalition and works with governments to accelerate climate action through three work streams. It supports governments to develop robust medium and long-term (2050) emissions reduction plans in line with the goals of the Paris Climate Agreement. Hence, option (d) is the correct answer.

Source: https://vajiramias.com/current-affairs/fridays-for-future- movement/5dba6cab1d5def24d8d75a0e/

Q41. With reference to the phosphorescence phenomenon, consider the following statements:

1. In it, after the radiation is removed, light emission stops immediately.

2. A phosphorescent material immediately re-emit the radiation it absorbs.

Which of the statements given above is/are correct? a. 1 only VAJIRAM & RAVI b. 2 only c. Both 1 and 2 d. Neither 1 nor 2

Answer: d

Explanation:

Statement 1 is incorrect:

Phosphorescence is emission of light from a substance exposed to radiation and persisting as an afterglow after the exciting radiation has been removed. Unlike fluorescence, in which the absorbed light is spontaneously emitted about 10-8 second after excitation, phosphorescence requires additional excitation to produce radiation and may last from about 10-3 second to days or years, depending on the circumstances.

Statement 2 is incorrect:

Unlike fluorescence, a phosphorescent material does not immediately re-emit the radiation it absorbs. The slower time scales of the re-emission are associated with "forbidden" energy state transitions in quantum mechanics. As these transitions occur very slowly in certain materials, absorbed radiation is re- emitted at a lower intensity for up to several hours after the original excitation.

Everyday examples of phosphorescent materials are the glow-in-the-dark toys that glow after being charged with a bright light such as in any normal reading or room light. Typically, the glow slowly fades out, sometimes within a few minutes or up to a few hours in a dark room.

The Delhi-based National Physical Laboratory (CSIR-NPL) has recently synthesised a novel security ink that emits intense red colour when exposed to 254 nm wavelength UV and emits green colour soon after the UV source is turned off. The emission of red is due to fluorescence while green is due to phosphorescence phenomenon. Both red and green can be clearly seen with the naked eye under ambient conditions. The red colour is emitted at 611 nm wavelength while the green is emitted at 532 nm. The ink has the potential to be used as a security feature on currency notes and passports. This is the first report of an ink that contains two pigments that emit different colours at very different wavelengths when exposed to UV light of a particular wavelength.

Source: https://vajiramias.com/current-affairs/phosphorescence/5da6a5af1d5def39fef3a3be/

Q42. With reference to the KHON Ramlila, consider the following statements:

1. It is a masked form of Ramlila art of Sri Lanka.

2. It is included in the list of UNESCO’s Intangible cultural heritage.

Which of the statements given above is/are correct? a. 1 only b. 2 only VAJIRAM & RAVI c. Both 1 and 2 d. Neither 1 nor 2

Answer: b

Explanation:

Statement 1 is incorrect:

KHON Ramlila is a masked form of Ramlila art of Thailand. It’s a form of masked dance depicting the scenes of Ramlila. It has no dialogues and background voices narrate the whole story of Ramayana. It is also famous for its beautiful attire and golden masks.

Statement 2 is correct:

It is included in the list of UNESCO’s Intangible cultural heritage. The Ramakien (literally ‘Glory of Rama’) is Thailand's national epic, derived from the Hindu epic Ramayana.

Source: https://vajiramias.com/current-affairs/khon-ramlila/5da7d9f71d5def4029dbb696/

Q43. With reference to the 'South Asia Economic Focus’, consider the following statements:

1. It is released by the World Economic Forum (WEF).

2. Bhutan has become the second-fastest growing economy in South Asia.

Which of the statements given above is/are correct? a. 1 only b. 2 only c. Both 1 and 2 d. Neither 1 nor 2

Answer: d

Explanation:

Statement 1 is incorrect:

The South Asia Economic Focus is a biannual economic update presenting recent economic developments and a near-term economic outlook for South Asia. World Bank released its latest report titled 'South Asia Economic Focus, Making (De) Centralization Work,' a twice-a-year regional economic update. In line with a global downward trend, growth in South Asia is projected to slow to 5.9% in 2019, down 1.1 percentage points from April 2019 estimates.

Statement 2 is incorrect:

Bangladesh has become the second-fastest growing economy in South Asia after Bhutan. In Bangladesh, the real GDP growth is estimated at 8.1 per cent in 2019, up from 7.9 per cent in 2018. The country's growth is projected at 7.2 per cent in 2020 and 7.3 per cent in 2021. In India, growth is projected to fall to VAJIRAM & RAVI

6.0 this fiscal year. Growth is then expected to gradually recover to 6.9 per cent in fiscal year 2021 and to 7.2 per cent in the following year. Pakistan's growth rate is projected to deteriorate further to a mere 2.4 per cent this fiscal year, as monetary policy remains tight, and the planned fiscal consolidation will compress domestic demand, it said. Decentralisation in South Asia has yet to deliver on its promises and, if not properly managed, can degenerate into fragmentation.

Source: https://vajiramias.com/current-affairs/south-asia-economic-focus-making-de-centralization- work/5da7d8c21d5def4028c550d7/

Q44. Reserve Bank India (RBI) has recently decided to offer on-tap authorisation for which of the following payment systems?

1. Bharat Bill Payment Operating Unit (BBPOU).

2. Trade Receivables Discounting System (TReDS).

3. White Label ATMs (WLAs).

Select the correct answer using the code given below: a. 1 only b. 1 and 2 only c. 2 and 3 only d. 1, 2 and 3

Answer: d

Explanation:

Reserve Bank of India (RBI) has recently issued guidelines on ‘on tap' authorisation of payment systems. In order to benefit from diversification of risk as also to encourage innovation and competition, RBI has decided to offer on-tap authorisation for the following payment systems –

o Bharat Bill Payment Operating Unit (BBPOU). o Trade Receivables Discounting System (TReDS). o White Label ATMs (WLAs). • Minimum net-worth criteria: o For entities desirous to function, operate, or provide platforms for BBPOU, they should have a 100 crore rupees net worth and should be maintained at all times. o In case of TReDS, the minimum paid up equity capital should be 25 crore rupees. o The minimum net worth for entities desirous of entering the WLA segment should be 100 crore rupees. • The KYC requirements for retail payment systems shall be as per the Master Directions on Know Your Customer (KYC) issued by the Department of Banking Regulation (DBR), RBI. Hence, option (d) is the correct answer.

Value Addition: VAJIRAM & RAVI

An ‘on-tap’ facility would mean the RBI will accept applications and grant license for payment systems throughout the year, at any time, subject to the fulfilment of the set conditions.

Source: https://vajiramias.com/current-affairs/on-tap-authorisation-of-payment- systems/5da7d9a11d5def4028c55102/

Q45. What is common to the places known as Sirumugai, Chanderi and Sambalpur? a. Silk Sarees b. Tropical rain forests c. Underground cave systems d. Water reservoirs

Answer: a

Explanation:

Sirumugai is a town in Coimbatore district in Tamil Nadu famous for silk saree production. Sirumugai is well known for Silk Sarees Bazaars.

Chanderi Silk Sarees are produced from chanderi cotton and silk cotton in Chanderi . The lightweight and shimmered look Chanderi Silk Sarees are listed under Geographical Indications in India.

Sambalpuri Silk Saree are manufactured in Sambalpur, Bargarh, Sonepur and Berhampur and listed under the Government of India’s Geographical Indications.

Hence, option (a) is the correct answer.

Source: https://vajiramias.com/current-affairs/sirumugai/5da928871d5def5b8772daba/

Q46. Mount Dilli, recently seen in news, located in which of the following state/ Union territory? a. Delhi NCR b. Maharashtra c. Kerala d. Karnataka

Answer: c

Explanation:

Mount Dilli (Mount Dilli) is a hill in Kerala with an elevation of 157 meters above sea level.

The (INA Ezhimala) is the initial officer training establishment of the Indian Navy and the Indian Coast Guard, located in Ezhimala, Kannur district, Kerala. Mount Dilli located within INA, Ezhimala premises. Dilli Series Seminar is conducted at INA during Autumn Term every year, with an aim to expose young trainees to the vibrant maritime history and arouse in them a curiosity to explore VAJIRAM & RAVI further. The theme of the sixth edition of the seminar was ‘Role of Sea Power in Shaping Nations’. Hence, option (c) is the correct answer.

Source: https://vajiramias.com/current-affairs/dilli-series-sea-power- seminar/5daa4f641d5def7649307ea0/

Q47. ‘Unicorn club’ sometimes seen in the news is related to: a. Startups valued at $1 billion and above. b. The United Kingdom’s European Union membership referendum. c. Movement in opposition to the possible expansion of permanent seats in the UNSC. d. None of the above.

Answer: a

Explanation:

In the venture capital industry, a unicorn refers to any tech startup company that reaches a $1 billion dollar market value as determined by private or public investment. The term was originally coined by Aileen Lee, founder of Cowboy Ventures. Hence, option (a) is the correct answer.

According to Hurun Global Unicorn List 2019, India, with 21 unicorns, has emerged as third largest ecosystems for more successful startups right behind China and US but ahead of Britain and Israel.

Source: https://vajiramias.com/current-affairs/hurun-global-unicorn-list- 2019/5daa439a1d5def7647cf9180/

Q48. What is ‘Spectroscopy’, recently seen in news? a. It is the technique of splitting light into its constituent wavelengths. b. It is the technical field of using microscopes to view objects and areas of objects that cannot be seen with the naked eye. c. It is the study of the relative spatial arrangement of atoms that form the structure of molecules and their manipulation. d. It is the experimental science of determining the arrangement of atoms in crystalline solids.

Answer: a

Explanation:

Spectroscopy is the technique of splitting light (or more precisely electromagnetic radiation) into its constituent wavelengths (a spectrum), in much the same way as a prism splits light into a rainbow of colours. Old style spectroscopy was carried out using a prism and photographic plates. These days, modern spectroscopy uses diffraction gratings to disperse the light, which is then projected onto CCDs (Charge Coupled Devices) similar to those used in digital cameras. Imaging Infrared Spectrometer (IIRS) is designed to measure light from the lunar surface in narrow spectral channels (bands). It has the ability VAJIRAM & RAVI to split and disperse reflected sunlight (and its emitted component) into these spectral bands. From the reflected solar spectrum, scientists will look for signatures, including of minerals. This will help map the lunar surface composition, which in turn will help us understand the Moon’s origin and evolution in a geologic context. Hence, option (a) is the correct answer.

Source: https://vajiramias.com/current-affairs/spectroscopy/5dabc0641d5def1891a5f339/

Q49. Bali Yatra, one of the biggest trade fairs of which one of the following states? a. Kerala b. Tamil Nadu c. Odisha d.

Answer: c

Explanation:

Bali Yatra, one of the biggest trade fairs of Odisha. Bali Jatra (also known as Bali Yatra and Boita Bandaṇa) literally means A Voyage to Bali. This festival is held in Odisha, in the city of Cuttack at Gadagadia Ghata of the Mahanadi river. The festival is celebrated every year from the day of Kartika Purnima according to the Odia Calendar, which comes around the end of October and November. It is celebrated to mark the day when ancient Sadhabas (Odia mariners) would set sail to distant lands of Bali, as well as Java, Sumatra and Sri Lanka for trade and cultural expansion. People of Odisha gather near banks of Mahanadi, , other river banks, ponds to float miniature toy boats, made of colored paper as a symbolic gesture of their ancestors' voyage. In Cuttack, Bali Jatra is celebrated annually as a large open fair near the Barabati Fort area with several cultural programs, toy stalls, different games and food stalls selling Odia delicacies. Hence, option (c) is the correct answer.

Source: https://vajiramias.com/current-affairs/bali-yatra/5dabc19a1d5def1892592492/

Q50. Sentinel-3 is an Earth observation satellite constellation developed by which one of the following Space Organisations? a. European Space Agency (ESA) b. National Aeronautics and Space Administration (NASA) c. Indian Space Research Organisation (ISRO) d. Japan Aerospace Exploration Agency (JAXA)

Answer: a

Explanation:

Sentinel-3 is an Earth observation satellite constellation developed by the European Space Agency (ESA) as part of the Copernicus Programme. The Sentinel-3 World Fires Atlas Prototype product has been developed by ESA over the southern countries of the ESA member states. It aims to provide continuity VAJIRAM & RAVI to ESA ATSR World Fire Atlas (operating from June 1995 to March 2012) taking on board all the improvements of Sentinel-3 SLSTR instrument. Hence, option (a) is the correct answer.

Source: https://vajiramias.com/current-affairs/sentinel-3-world-fire- atlas/5db926251d5def24d9358e63/ Q51. Consider the following pairs:

Which of the following pairs is/are correctly matched? a. 3 only b. 1 and 2 only c. 2 and 3 only d. 1, 2 and 3

Answer: a

Explanation:

Pair (1) is not correctly matched: ChavangKut, the post-harvest festival of the Kuki-Chin-Mizo communities. It is one of the most important festivals of Kuki-Chin-Mizo communities. In , and and other parts of country, the festival is organized every year on the first day of November month. Chavang stands for ‘autumn’ and Kut mean ‘harvest’. The festival is observed to mark the end of the harvesting season and also to thank deities for a good harvest season.

Pair (2) is not correctly matched: Mamita of is performed on the occasion of Mamita festival, which is harvest festival of Tripuri People.

Pair (3) is correctly matched: Behdiengkhlam festival is a traditional festival celebrated after sowing is done seeking a good harvest and to drive away plague and diseases.

Hence, option (a) is the correct answer.

Source: https://vajiramias.com/current-affairs/chavang-kut/5dbcda0f1d5def2b9fb108d0/

Q52. With reference to the UNESCO Creative Cities Network (UCCN), consider the following statements:

1. It is a network of cities which are thriving, active centres of cultural activities in their respective countries, created in 1980. VAJIRAM & RAVI

2. UNESCO has recently designated Mumbai as a member of UNESCO Creative Cities Network (UCCN) in the field of FILM and Hyderabad in the field of Gastronomy.

Which of the statements given above is/are correct? a. 1 only b. 2 only c. Both 1 and 2 d. Neither 1 nor 2

Answer: b

Explanation:

Statement 1 is incorrect:

UNESCO Creative Cities Network (UCCN), created in 2004, is a network of cities which are thriving, active centres of cultural activities in their respective countries. The member cities that form part of the Network come from all continents and regions with different income levels and populations. They work together towards a common mission: placing creativity and the creative economy at the core of their urban development plans to make cities safe, resilient, inclusive and sustainable, in line with the United Nations 2030 Agenda for Sustainable Development.

Statement 2 is correct:

o UNESCO has recently designated Mumbai as a member of UNESCO Creative Cities Network (UCCN) in the field of FILM and Hyderabad in the field of GASTRONOMY. The Indian cities which were recognized in the past includes Jaipur for Crafts and Folk Arts(2015), Varanasi for Creative city of Music (2015) and Chennai for Creative city of Music(2017).UCCN now counts a total of 246 cities.

Hence, option (b) is the correct answer.

Value Addition: The 7 categories for recognition under UCCN are as follows-

o Crafts and Folk Arts o Design o Film o Gastronomy o Music o Media Arts o Literature

Source: https://vajiramias.com/current-affairs/unesco-creative-cities-network- uccn/5dbce0681d5def2b9e6d615a/ VAJIRAM & RAVI

Q53. Consider the following statements:

1. It borders Peru to the north and Bolivia to the northeast.

2. The Andes Mountains are located on the eastern border and the Pacific Ocean to the west.

3. The arid Atacama Desert located in this country.

4. It is the world's southernmost country that is geographically on the mainland.

Which of the following countries is best described in the above given statements? a. Chile b. Brazil c. Argentina d. Colombia

Answer: a

Explanation:

Chile is a South American country. It borders Peru to the north, Bolivia to the northeast, Argentina to the east, and the Drake Passage in the far south. The Andes Mountains are located on the eastern border VAJIRAM & RAVI and the Pacific Ocean to the west. The Drake Passage is the body of water between South America's Cape Horn and the South Shetland Islands of Antarctica. Cape Horn is the southernmost headland of the Tierra del Fuego archipelago of southern Chile. The arid Atacama Desert in northern Chile contains great mineral wealth, principally copper and lithium. It is the world's southernmost country that is geographically on the mainland. Its Capital is Santiagoa.

Hence, option (a) is the correct answer.

Source: https://vajiramias.com/current-affairs/chile/5dbcdffa1d5def2b9e6d614c/

Q54. In the context of the electoral bonds, consider the following statements:

1. The Punjab National Bank is the only authorized bank to sell them.

2. Buyers can donate the bonds to a party of their choice which has to be cashed by the party through its verified account within 15 days.

Which of the statements given above is/are correct? a. 1 only b. 2 only c. Both 1 and 2 d. Neither 1 nor 2

Answer: b

Explanation:

Statement 1 is incorrect:

Electoral bonds are interest-free bearer bonds that are used to donate money anonymously to political parties. The State Bank of India (SBI) is the only bank authorised to sell them. The bonds are issued in multiples of Rs 1,000, Rs 10,000, Rs 1 lakh, Rs 10 lakh and Rs 1 crore which can be bought by Indian citizens or companies.

Statement 2 is correct:

Buyers can donate the bonds to a party of their choice and these can then be cashed by the party through its verified account within 15 days. The intention behind the scheme is to protect the identity of donors who fear adverse consequences for contributing to one political party, as stated by the government. However, the Election Commission and civil society have opposed the scheme and the opacity it has brought into political funding. A petition challenging the scheme is pending with the Supreme Court.

Hence, option (b) is the correct answer.

Source: https://vajiramias.com/current-affairs/electoral-bonds/5dbcdf691d5def2b9e6d6129/ VAJIRAM & RAVI

Q55. Which of the following are stated criteria to qualify as a dwarf planet as set by the International Astronomical Union?

1. It orbits around the Sun and it is not a moon of any other planet.

2. It has enough mass so that its own gravity pulls it into a roughly spherical shape.

Select the correct answer using the code given below: a. 1 only b. 2 only c. Both 1 and 2 d. Neither 1 nor 2

Answer: c

Explanation:

There are officially five dwarf planets in our Solar System. The most famous is Pluto, downgraded from the status of a planet in 2006. The other four, in order of size, are Eris, Makemake, Haumea and Ceres. Now, there is a claimant for a sixth dwarf planet called Hygiea, it has so far been taken to be an asteroid. It lies in the asteroid belt between Mars and Jupiter. If it qualifies, Hygiea will be the smallest dwarf planet in the Solar System. The International Astronomical Union sets four criteria for a dwarf planet:

• it orbits around the Sun, it is not a moon, and it has not cleared the neighbourhood around its orbit. • The fourth requirement is that it has to have enough mass that its own gravity pulls it into a roughly spherical shape.

According to the new study, VLT observations now show Hygiea satisfied that condition, too. This is the first time astronomers have observed Hygiea in high resolution to study its surface and determine its shape and size.

Hence, option (c) is the correct answer.

Source: https://vajiramias.com/current-affairs/hygiea/5dbe535b1d5def2b9a0365c7/

Q56. With reference to the Budapest Convention, consider the following statements:

1. It is also known as the Convention on Cybercrime of the Council of Europe.

2. India has been a permanent member since its adoption in 2001.

Which of the statements given above is/are not correct? a. 1 only b. 2 only c. Both 1 and 2 VAJIRAM & RAVI d. Neither 1 nor 2

Answer: b

Explanation:

Statement 1 is correct:

The Convention on Cybercrime of the Council of Europe (CETS No.185) is also known as the Budapest Convention on Cybercrime or the Budapest Convention. It is the first international treaty seeking to address cybercrime by harmonizing national laws, improving investigative techniques, and increasing cooperation among nations. The following offences are defined by the Convention: illegal access, illegal interception, misuse of devices, computer-related fraud, offences related to child pornography, and offences related to copyright and neighbouring rights.

Statement 2 is incorrect:

It was opened for signature in Budapest in 2001 and it entered into force in 2004. It was drawn up by the Council of Europe with the active participation of the Council of Europe's observer states Canada, Japan, Philippines, South Africa and the United States. As of September 2019, 64 states have ratified the convention. Brazil and India have declined to adopt the Convention on the grounds that they did not participate in its drafting. Russia opposes the Convention, stating that adoption would violate Russian sovereignty.

Hence, option (b) is the correct answer.

Source: https://vajiramias.com/current-affairs/budapest-convention/5dd89d9f1d5def2fe03636a2/

Q57. Which of the following best describes ‘Glyphosate’, recently seen in the news? a. Compound present in herbicides. b. Newly found Comet c. Mini Black Hole d. Gusting winds in northern California

Answer: a

Explanation:

First developed in 1970, glyphosate is scientifically N-(phosphonomethyl) glycine under the IUPAC system of nomenclature. It is applied to the leaves of plants to kill weeds. It is widely used in India, too. This weed-killer in India goes by various brand names, including Roundup, Glycel, and Brake. In 2015, the Word Health Organization’s International Agency for Research on Cancer (IARC) published a study that found glyphosate is “probably carcinogenic to humans”.

Hence, option (a) is the correct answer.

Source: https://vajiramias.com/current-affairs/glyphosate/5dbf8cd01d5def2ba0b91152/ VAJIRAM & RAVI

Q58. ‘YuWaah’ Generation Unlimited in India, recently in the news, is an initiative of a. UNICEF b. UNESCO c. World Bank d. World Trade Organisation (WTO)

Answer: a

Explanation:

UNICEF has launched ‘YuWaah’ Generation Unlimited in India on 1.11.2019. According to UNICEF, Generation Unlimited, called YuWaah in India, is a multi-stakeholder alliance which aims to facilitate youth to gain relevant skills for productive lives and the future of work. The target age group of YuWaah includes adolescent girls and boys. Its key mission is to promote access to foundational, transferable and 21st century skills for youth inside and outside formal education systems, which includes defining foundational skills, life skills and flexible learning and identifying and scaling impactful delivery models. YuWaah intends to create platforms to guide youth to market opportunities (career guidance, mentorship, internships, apprenticeships) and facilitate integration of career guidance in school education.

Hence, option (a) is the correct answer.

Source: https://vajiramias.com/current-affairs/yuwaah/5de0a0ca1d5def35c37e18ba/

Q59. With reference to the “Global Microscope on Financial Inclusion 2019”, consider the following statements:

1. It was released by World Economic Forum (WEF).

2. It accords India as the fifth most conducive environment among emerging countries for inclusive finance.

Which of the statements given above is/are correct? a. 1 only b. 2 only c. Both 1 and 2 d. Neither 1 nor 2

Answer: b

Explanation:

Statement 1 is incorrect: VAJIRAM & RAVI

The Economist Intelligence Unit (EIU) released a report titled the 2019 Global Microscope on Financial Inclusion. The Global Microscope for Financial Inclusion, now in its 12th year, is a benchmarking index that assesses the enabling environment for financial access in 55 countries.

The EIU’s analysis takes four basic parameters into account:

(1) whether non-banks can issue e-money, (2) the presence of financial service agents, (3) proportionate customer due diligence and (4) effective financial consumer protection.

Statement 2 is correct:

Key findings are- Of the 55 countries assessed, only 4 countries - Colombia, India, Jamaica and Uruguay - scored perfectly across all four parameters. India has the fifth most conducive environment among emerging countries for inclusive finance. Only Columbia, Peru, Uruguay and Mexico are ahead of India.

Hence, option (b) is the correct answer.

Source: https://vajiramias.com/current-affairs/global-microscope-for-financial-inclusion- 2019/5dc0f9501d5def70b630e7df/

Q60. What is ‘ICEDASH’, recently seen in the news? a. It is an Ease of Doing Business monitoring dashboard of the Indian Customs. b. It is a group of bacteria that can cause foodborne illnesses. c. It is a new mineral, which has been discovered inside a diamond. d. None of the above.

Answer: a

Explanation:

ICEDASH is an Ease of Doing Business monitoring dashboard of the Indian Customs helping the public see the daily Customs clearance times of import cargo at various ports and airports. It will help the businesses in comparing clearance times across ports and thus plan their logistics accordingly. This dashboard has been developed by Central Board of Indirect Taxes and Customs (CBIC) in collaboration with National Informatics Centre.

Hence, option (a) is the correct answer.

Source: https://vajiramias.com/current-affairs/icedash-atithi/5dc0fc081d5def70bbe3e92a/

Q61. With reference to the UN Environment Programme’s (UNEP) annual Emissions Gap Report for the year 2019, consider the following statements:

1. Greenhouse gas emissions have risen 1.5 per cent per year over the last decade.

2. China is the highest producer of greenhouse gases when measured on a per capita basis. VAJIRAM & RAVI

Which of the statements given above is/are correct? a. 1 only b. 2 only c. Both 1 and 2 d. Neither 1 nor 2

Answer: a

Explanation:

Statement 1 is correct:

UN Environment Programme (UNEP) recently released its annual Emissions Gap Report for the year 2019.Greenhouse gas emissions have risen 1.5 per cent per year over the last decade. Emissions in 2018, including from land-use changes such as deforestation, hit a new high of 55.3 gigatonnes of CO2 equivalent. Statement 2 is incorrect:

The United States is the highest producer of greenhouse gases when measured on a per capita basis. Overall, China is the world’s largest producer. India is the third-largest emitter behind the United States and China respectively. India’s per capita emissions, however, are significantly below the United States, China and many others. Each year, the Emissions Gap Report assesses the gap between anticipated emissions in 2030 and levels consistent with the 1.5°C and 2°C targets of the Paris Agreement. For the first time, it looks at how large annual cuts would need to be from 2020 to 2030 to stay on track to meeting the Paris goals.

Hence, option (a) is the correct answer.

Source: https://vajiramias.com/current-affairs/emissions-gap-report- 2019/5ddf35db1d5def35c67e6cb9/

Q62. In which of the following states does river Feni originates? a. Assam b. Mizoram c. d. Tripura

Answer: d

Explanation:

Feni River is a trans-boundary river that originates in Tripura and flows through Sabroom town and then enters Bangladesh. Muhuri River, also called Little Feni, from joins it near its mouth. VAJIRAM & RAVI

The Union Cabinet has recently given ex-post facto approval for the MoU between India and Bangladesh on withdrawal of 1.82 cusec of water from by India for drinking water supply scheme for Sabroom town Tripura, India. Hence, option (d) is the correct answer.

Source: https://vajiramias.com/current-affairs/feni-river/5dc3a4971d5def70b4559f3c/

Q63. Consider the following statements:

1. Planet Venus rotates in a direction opposite to that for most planets.

2. Planet Saturn is full of solid surface features which emits only high-frequency radio patterns that are blocked by Earth’s atmosphere.

Which of the statements given above is/are correct? a. 1 only b. 2 only c. Both 1 and 2 d. Neither 1 nor 2

Answer: a

Explanation:

Statement 1 is correct:

Venus; Obscured by clouds, it does not present a readily visible surface feature, such as a crater, which could have been a reference point for measuring its rotation period. In 1963, when radar observations broke through the cloud cover, Venus revealed that it rotates in a direction opposite to that for most planets. Studies based on the Magellan spacecraft’s observations concluded that the correct rotation period is 243.0185 days, with an uncertainty of about 9 seconds. Subsequent measurements have, however, given inconsistent values.

Statement 2 is incorrect:

Saturn; A gas giant, by definition, it has no solid surface features for scientists to track. For Jupiter, scientists figured out the rotation period by observing patterns in radio signals from there. Saturn has defied such attempts. It emits only low-frequency radio patterns that are blocked by Earth’s atmosphere.

Hence, option (a) is the correct answer.

Source: https://vajiramias.com/current-affairs/how-long-is-a-day-on-each- planet/5dc4ef3e1d5def70b9ea2094/

Q64. Consider the following pairs: VAJIRAM & RAVI

Which of the following pairs is/are correctly matched? a. 1 only b. 1 and 2 only c. 2 and 3 only d. 1, 2 and 3

Answer: a

Explanation:

Pair (1) is correctly matched: Bodos are the single largest tribal community in Assam, making up over 5- 6 per cent of the state’s population. They have controlled large parts of Assam in the past. The four districts in Assam — Kokrajhar, Baksa, Udalguri and Chirang — that constitute the Bodo Territorial Area District (BTAD), are home to several ethnic groups. The Bodos have had a long history of separatist demands, marked by armed struggle.

Pair (2) is not correctly matched: The Apatanis are a tribal group of people living in the Ziro valley in the Lower Subansiri district of Arunachal Pradesh.

Pair (3) is not correctly matched: The Brus, also referred to as the Reangs, are spread across the north- eastern states of Tripura, Assam, Manipur, and Mizoram. In Tripura, they are recognised as a Particularly Vulnerable Tribal Group. They speak the Reang dialect of language which is locally referred to as KauBru. Their Hojagiri folk dance is well known all over the world. 'Buisu', not 'bihu' is the most popular festival of Reang tribes.

Hence, option (a) is the correct answer.

Source: https://indianexpress.com/article/explained/explained-what-is-the-bodoland-dispute-and- who-are-the-ndfb-6136083/

Q65. With reference to the Overseas Citizen of India (OCI), consider the following statements:

1. If an individual is registered as an OCI for a period of five years, s/he is eligible to apply for Indian citizenship.

2. An applicant is eligible for the OCI card if s/he, his parents or grandparents have ever been a citizen of Bangladesh and Pakistan only.

Which of the statements given above is/are correct? VAJIRAM & RAVI a. 1 only b. 2 only c. Both 1 and 2 d. Neither 1 nor 2

Answer: a

Explanation:

An Overseas Citizen of India, or OCI, is a category introduced by the government in 2005. Persons of Indian Origin (PIOs) of certain categories as specified in the Citizenship Act, 1955 are eligible for being OCI cardholders. Some of the benefits for PIO and OCI cardholders were different until 2015, when the government merged these two categories.

• Eligibility: o The Ministry of Home Affairs defines an OCI as a person who was a citizen of India on or after January 26, 1950; or was eligible to become a citizen of India on that date; or who is a child or grandchild of such a person, among other eligibility criteria. o An applicant is not eligible for the OCI card if he, his parents or grandparents have ever been a citizen of Pakistan or Bangladesh. • Benefits: o OCI cardholders can enter India multiple times, get a multipurpose lifelong visa to visit India, and are exempt from registering with Foreigners Regional Registration Office (FRRO) no matter how long their stay. o If an individual is registered as an OCI for a period of five years, he/she are eligible to apply for Indian citizenship. o OCI cardholders can open special bank accounts in India, they can buy non-farm property and exercise ownership rights and can also apply for a driver’s license and PAN card. So, only statement 1 is correct.

Hence, option (a) is the correct answer.

Source: https://vajiramias.com/current-affairs/overseas-citizen-of-india- oci/5dc640d61d5def70bbe4e2b0/

Q66. With reference to the ‘ozone hole’, consider the following statements:

1. It is a region in the stratosphere, directly above Antarctica built because of destruction of ozone layer.

2. The Montreal Protocol was aimed at phased elimination of chloroflurocarbons (CFCs), responsible for development of ozone hole.

Which of the statements given above is/are correct? a. 1 only VAJIRAM & RAVI b. 2 only c. Both 1 and 2 d. Neither 1 nor 2

Answer: c

Explanation:

Statement 1 is correct:

Ozone (chemically, a molecule of three oxygen atoms) is found mainly in the upper atmosphere, an area called stratosphere, between 10 and 50 km from the earth’s surface. By absorbing the harmful ultraviolet radiations from the sun, the ozone molecules eliminate a big threat to life forms on earth. UV rays can cause skin cancer and other diseases and deformities, in plants and animals. The ‘ozone hole’ is not really a hole. It is a region in the stratosphere, directly above Antarctica, where the concentration of ozone has been measured to become extremely low in certain months.

Statement 2 is correct:

Use of industrial chemicals like chloroflurocarbons (CFCs) in refrigeration, air-conditioning, foams, fire- extinguishers and solvents were the likely culprits. A 1989 Global Agreement, called Montreal Protocol, organised international consensus on phased elimination of these chemicals. Two years ago, an amendment to the Montreal Protocol cleared the way for a faster elimination of another set of similar compounds, called hydroflurocarbons, or HFCs, which were being used as temporary replacements for CFCs.

Hence, option (c) is the correct answer.

Source: https://vajiramias.com/current-affairs/ozone-hole/5dc796551d5def70b9ea910b/

Q67. With reference to the 'Special Window for funding stalled housing project’, consider the following statements:

1. The fund will be set up as a Category-II Alternate Investment Fund (AIF) debt fund registered with SEBI.

2. The total commitment of funds to be infused by the Government through the Special Window would be up to INR 10,000 crores.

Which of the statements given above is/are correct? a. 1 only b. 2 only c. Both 1 and 2 d. Neither 1 nor 2 Answer: c VAJIRAM & RAVI

Explanation:

Statement 1 is correct:

The Union Cabinet recently approved the establishment of a 'Special Window' fund to provide priority debt financing for the completion of stalled housing projects that are in the Affordable and Middle- Income Housing sector. This special window will provide last mile funding for housing projects which are stressed. This fund would provide relief to developers that require funding to complete a set of unfinished projects and consequently ensure delivery of homes to the home-buyers. The fund will be set up as a Category-II AIF (Alternate Investment Fund) debt fund registered with SEBI and would be professionally run.

Statement 2 is correct:

The Government shall act as the sponsor to the proposed fund and shall have the authority and responsibility as specified under SEBI (AIF) Regulations, 2012. Whereas, the investment manager is responsible for fund raising, investments and managing the fund team. The total commitment of funds to be infused by the Government through the Special Window would be up to INR 10,000 cr. The fund is seeking matching contributions from Banks, LIC and others to generate a total corpus of around INR 25,000 cr. For the first AIF under the Special Window, SBICAP Ventures Limited shall be engaged to be the Investment Manager.

Hence, option (c) is the correct answer.

Source: https://vajiramias.com/current-affairs/special-window-for-funding-stalled-housing- project/5dc8ceab1d5def0775ac4769/

Q68. With reference to the Innovations for Defence Excellence (iDEX), consider the following statements:

1. It will be funded and managed by the NITI Aayog and the DRDO.

2. It aims at creation of an ecosystem to foster innovation in Defence and Aerospace by engaging multiple stakeholders.

Which of the statements given above is/are correct? a. 1 only b. 2 only c. Both 1 and 2 d. Neither 1 nor 2

Answer: b

Explanation:

Innovations for Defence Excellence (iDEX) was launched by the Government of India in 2018. It primarily aims at creation of an ecosystem to foster innovation in Defence and Aerospace by engaging Industries including MSMEs, start-ups, individual innovators, R&D institutes & academia, and provide them VAJIRAM & RAVI grants/funding and other support to carry out R&D which has good potential for future adoption for Indian defence and aerospace needs. iDEX will be funded and managed by a ‘Defence Innovation Organization (DIO)’ which has been formed as a ‘not for profit’ company as per Section 8 of the Companies Act 2013 by the two founder members i.e. Defence Public Sector Undertakings (DPSUs) - HAL & BEL. iDEX will function as the executive arm of DIO, carrying out all the required activities while DIO will provide high level policy guidance to iDEX. Hence only statement 2 is correct.

Hence, option (b) is the correct answer.

Source: https://vajiramias.com/current-affairs/innovations-for-defence-excellence- idex/5dc8cf621d5def07788fac83/

Q69. Which of the following states has the disputed territory of Kalapani, as per the new political map of India? a. Uttarakhand b. Sikkim c. d. Bihar

Answer: a

Explanation:

Kalapani is a 372-sq km area at the China-Nepal-India tri-junction. India claims Kalapani as a part of Uttarakhand while Nepal depicts the area in its map. The border dispute flared up again recently after India released its new political map, following the reorganisation of J&K, showing the area as its own. According to the Sugauli treaty signed between Nepal and British India in 1816, the Mahakali river that runs through the Kalapani area is the boundary between the two countries. However, British surveyors subsequently showed the origin of the river, which has many tributaries, at different places. While Nepal claims that the river west of the disputed territory is the main river and so Kalapani falls in its territory, India claims a different origin and includes the area in its territory. Hence, option (a) is the correct answer.

Source: https://vajiramias.com/current-affairs/kalapani/5dc8d1e61d5def077c475900/

Q70. With reference to the JalJeevan Mission (JJM), consider the following statements:

1. It aims at providing Functional Household Tap Connections (FHTCs) in rural areas by 2024.

2. Ministry of Rural Development is the implementing agency for the mission.

Which of the statements given above is/are correct? a. 1 only b. 2 only c. Both 1 and 2 VAJIRAM & RAVI d. Neither 1 nor 2

Answer: a

Explanation:

The JalJeevan Mission (JJM) aims at providing Functional Household Tap Connections (FHTCs) in rural areas by 2024. It was announced by the Prime Minister on 73rd Independence Day. The Government of India and State Governments will work in partnership to further this goal. Approx. 14.60 crores rural households of the country will be provided with FHTCs. The provisional cost estimate for this mission is Rs 3.60 lakh Crores. Department of Drinking Water & Sanitation is the implementing Department for the mission. Hence only statement 1 is correct.

Hence, option (a) is the correct answer.

Source: https://vajiramias.com/current-affairs/jal-jeevan-mission-jjm/5dca2b731d5def077c4799ba/

Q71. With reference to the International Treaty of Plant Genetic Resources for Food and Agriculture (ITPGRFA), consider the following statements:

1. It is a comprehensive international agreement for ensuring food security as well as the fair and equitable benefit sharing arising from its use.

2. The treaty was negotiated by the World Trade Organisation (WTO).

Which of the statements given above is/are correct? a. 1 only b. 2 only c. Both 1 and 2 d. Neither 1 nor 2

Answer: a

Explanation:

Statement 1 is correct:

International Treaty of Plant Genetic Resources for Food and Agriculture (ITPGRFA) is also known as Seed Treaty. It is a comprehensive international agreement for ensuring food security through the conservation, exchange and sustainable use of the world's Plant genetic resources for food and agriculture (PGRFA), as well as the fair and equitable benefit sharing arising from its use. It also recognizes farmers' rights, subject to national laws.

Statement 2 is incorrect:

The Treaty establishes the Multilateral System of Access and Benefit-sharing to facilitate plant germplasm exchanges and benefit sharing through Standard Material Transfer Agreement (SMTA). The treaty was negotiated by the Food and Agriculture Organization of the United Nations (FAO) VAJIRAM & RAVI

Commission on Genetic Resources for Food and Agriculture (CGRFA) and since 2006 has its own Governing Body under the aegis of the FAO.

Hence, option (a) is the correct answer.

Source: https://vajiramias.com/current-affairs/international-treaty-of-plant-genetic-resources-for- food-and-agriculture-itpgrfa/5dca2bd51d5def077c4799c7/

Q72. With reference to the Credit ratings, consider the following statements:

1. It is assigned to debt and equity instruments by a Credit Rating agency (CRA).

2. Credit rating agencies are regulated by SEBI under the SEBI (Credit Rating Agencies) Regulations, 1999.

Which of the statements given above is/are correct? a. 1 only b. 2 only c. Both 1 and 2 d. Neither 1 nor 2

Answer: b

Explanation:

Credit ratings are assigned to debt instruments (not to equity instrument) by a Credit Rating agency (CRA). Rating is denoted by a simple alphanumeric symbol, for e.g. AA+, A-, etc. Rating indicates that whether the issuer company can repay its debt obligation in full and on time. The Credit rating serves 2 main purpose: 1) Borrowing cost: It Influences the borrowing cost of country in international market and 2) Investment: Credit rating Influences foreign investors decision to invest I.E. by seeing this, the investor decides whether to buy, hold, or sell a debt instrument.

A credit rating agency is an entity which assesses the ability and willingness of the issuer company for timely payment of interest and principal on a debt instrument. Credit rating agencies are regulated by SEBI under the SEBI (Credit Rating Agencies) Regulations, 1999. Some of the Global CRAs are: Fitch, Moody, S&P. Some of the Indian CRAs are: Credit Analysis & Research Ltd. (CARE), Credit rating information services of India (CRISIL), Investment and credit rating agencies (ICRA) etc. Hence only statement 2 is correct.

Hence, option (b) is the correct answer.

Source: https://vajiramias.com/current-affairs/credit-ratings/5dca2ccf1d5def077a00cc85/

Q73. With reference to a recent study published in “The Lancet Global Health on anaemia among men in India”, consider the following statements:

1. Among age groups, men in the group 20-34 years had the lowest probability of having anaemia. VAJIRAM & RAVI

2. Among the states, the highest prevalence of any anaemia is in Odisha, with 32.9% of the men reporting it.

Which of the statements given above is/are not correct? a. 1 only b. 2 only c. Both 1 and 2 d. Neither 1 nor 2

Answer: b

Explanation:

A recent study published in The Lancet Global Health looked at anaemia among men in India.

Key Findings:

• Nearly a quarter of men in the age group 15-54 had some form of anaemia. • Cases among men ranged from moderate or severe (5.1%) to severe anaemia (0.5%). • Among age groups, men in the group 20-34 years had the lowest probability of having anaemia, while actual prevalence was lowest in the age group 50-54, at 7.8%. The prevalence was higher for younger age groups. • Among men with anaemia, 21.7% had moderate or severe anaemia; among women with anaemia, 53.2% had moderate or severe anaemia. • Among the states, the highest prevalence of any anaemia was in Bihar, with 32.9% of the men reporting it. The lowest prevalence among men was in Manipur (9.19%).Hence only statement 1 is correct.

Hence, option (b) is the correct answer.

Value Addition: Anaemia is a condition in which the number of red blood cells or their oxygen-carrying capacity is insufficient to meet physiological needs. Anaemia in men can cause fatigue, lethargy, creates difficulty in concentrating, thereby reducing the quality of life and decreasing economic productivity. Source: https://vajiramias.com/current-affairs/anaemia-among-men/5dca2d781d5def077a00ccaa/

Q74. Consider the following pairs:

VAJIRAM & RAVI

Which of the following pairs is/are correctly matched? a. 1 only b. 1 and 2 only c. 2 and 3 only d. 1, 2 and 3

Answer: a

Explanation:

Pair (1) is correctly matched: A is a community-owned rainwater storage mainly used for harnessing water resources in the state of Haryana, Rajasthan, Punjab, western Uttar Pradesh. It collects and stores water throughout the year, to be used for the purpose of recharging the groundwater in the nearby water wells, washing, bathing and drinking by humans and cattle.

Pair (2) is not correctly matched: Paar is a common water harvesting practice in the western Rajasthan region. It is a common place where the rainwater flows from the agar (catchment) and in the process percolates into the sandy soil. Rainwater harvested through PAAR technique is known as Patalipaani.

Pair (3) is not correctly matched: Bhitada village, Jhabua district of Madhya pradesh developed the unique pat system. This system was devised according to the peculiarities of the terrain to divert water from swift-flowing hill streams into irrigation channels called pats.

Hence, option (a) is the correct answer.

Source: https://vajiramias.com/current-affairs/johads/5ddc90671d5def55aa7714f0/

Q75. National Education Day is observed to commemorate the Birth Anniversary of a. Maulana Abul Kalam Azad b. Sarvepalli Radhakrishnan c. A. P. J. Abdul Kalam d. C. V. Raman

Answer: a

Explanation:

National Education Day is celebrated on November 11. This day is observed every year in India to commemorate the Birth Anniversary of Maulana Abul Kalam Azad, the first Education Minister of independent India, who served from 15 August 1947 until 2 February 1958. As Chairman of the Central Advisory Board of Education, he gave thrust to adult literacy, universal primary education, free and compulsory for all children up to the age of 14, girls education, and diversification of secondary education and vocational training. He oversaw the setting up of the Central Institute of Education, Delhi, which later became the Department of Education of the University of Delhi as "a research centre for VAJIRAM & RAVI solving new educational problems of the country". Under his leadership, the Ministry of Education established the first Indian Institute of Technology in 1951 and the University Grants Commission in 1953. He also laid emphasis on the development of the Indian Institute of Science, Bangalore and the Faculty of Technology of the Delhi University.

Hence, option (a) is the correct answer.

Source: https://vajiramias.com/current-affairs/national-education-day/5dcb71471d5def077c47d6aa/

Q76. With reference to the Suranga Bawadi, consider the following statements:

1. it was recently included in the World Monument Watch list for 2020 by UNESCO.

2. It is an integral part of the ancient Karez system of supplying water built during AdilShahi era in Karnataka.

Which of the statements given above is/are correct? a. 1 only b. 2 only c. Both 1 and 2 d. Neither 1 nor 2

Answer: b

Explanation:

A New York-based NGO has recently included Suranga Bawadi in the World Monument Watch list for 2020 along with 24 other monuments from across the world. Suranga Bawadi is an integral part of the ancient Karez system of supplying water through subterranean tunnels built during AdilShahi era in Karnataka. The AdilShahis built the Karez underground water system in the 16th century to supply water to Vijayapura in Karnataka. The monument has been selected under the “Ancient Water System of the ” by the World Monuments Fund (A New York-based NGO), which monitors restoration of ancient monuments across the globe. With this, the Suranga Bawadi is expected to get funds for restoration within the next two years. So, only statement 2 is correct.

Hence, option (b) is the correct answer.

Additional info: Karez system was built by Bahmani kings in 15th Century by the Bahmani kings in Bidar, Gulbarg and Bijapur in Karnataka and also in Burhanpur in Madhya Pradesh. Kariz is nothing but the Underground canals, built to underground water streams which are meant to provide drinking water to civilian settlements and garrison inside the Bidar fort. This system was necessary in a city like Bidar where the soil was rocky and drilling wells to accommodate drinking water was difficult.

Source: https://vajiramias.com/current-affairs/suranga-bawadi/5dcb71b31d5def077a0108b4/ VAJIRAM & RAVI

Q77. Which of the following comes under the definition of “Public Authority” as stated under the Right to Information Act, 2005?

1. Food Corporation of India

2. Unaided religious trusts

3. Office of Chief Justice of India

Select the correct answer using the code given below: a. 1 and 3 only b. 3 only c. 1 and 2 only d. 1, 2 and 3 only

Answer: a

Explanation:

Public authorities are the repository of information which the citizen have right to have under the Right to Information Act, 2005. As defined in the Act, a “ Public authority ” is any authority or body or institution of self government established or constituted by or under the Constitution; or by any other law made by the Parliament or a State Legislature; or by notification issued or order made by the Central Government or a State Government. Bodies owned, controlled or substantially financed by the Central Government substantially financed by the Central Government or State Government also fall within the definition of public authority. The financing of the body or the NGO by the Government may be direct or indirect.

Recently the Supreme Court held that office of Chief Justice of India is a public authority under Right to Information (RTI) Act.

Hyderabad High Court has recently held that religious institutions like temples, churches and mosques that are not funded by the government do not fall within the purview of the Right to Information Act- 2005 (RTI).

Hence, option (a) is the correct answer.

Source: https://vajiramias.com/current-affairs/right-to-information-rti/5dccbb351d5def077656a62b/

Q78. With reference to the Pneumonia, consider the following statements:

1. It is a form of acute respiratory infection that affects the lungs.

2. According to WHO, it is the single largest infectious cause of death in children worldwide.

Which of the statements given above is/are correct? a. 1 only VAJIRAM & RAVI b. 2 only c. Both 1 and 2 d. Neither 1 nor 2

Answer: c

Explanation:

Pneumonia is a form of acute respiratory infection that affects the lungs. The lungs are made up of small sacs called alveoli, which fill with air when a healthy person breathes. When an individual has pneumonia, the alveoli are filled with pus and fluid, which makes breathing painful and limits oxygen intake. Pneumonia is caused by a number of infectious agents, including viruses, bacteria and fungi.

Pneumonia can spread in a number of ways via air-borne droplets from a cough or sneeze or blood, especially during and shortly after birth. Pneumonia can be prevented by immunization, adequate nutrition, and by addressing environmental factors. Pneumonia caused by bacteria can be treated with antibiotics. According to WHO, Pneumonia is the single largest infectious cause of death in children worldwide. Pneumonia killed 808 694 children under the age of 5 in 2017, accounting for 15% of all deaths of children under five years old. The WHO and UNICEF integrated Global action plan for pneumonia and diarrhoea(GAPPD) aims to accelerate pneumonia control with a combination of interventions to protect, prevent, and treat pneumonia in children Hence both statements are correct.

Hence, option (c) is the correct answer.

Source: https://vajiramias.com/current-affairs/pneumonia/5dccb9c81d5def077c481ac7/

Q79. With reference to the International Criminal Court (ICC), consider the following statements:

1. The Court's official seat is in the Hague, Netherlands, but its proceedings may take place anywhere.

2. While it is not a United Nations organization, the Court has a cooperation agreement with the United Nations.

Which of the statements given above is/are correct? a. 1 only b. 2 only c. Both 1 and 2 d. Neither 1 nor 2

Answer: c

Explanation:

International Criminal Court (ICC) is an intergovernmental international tribunal for Criminal prosecution of individuals for four main crimes namely Genocide, War crimes, Crimes against humanity and Crime of aggression. VAJIRAM & RAVI

It was established in 2002 by Rome statue. Headquarters is in Hague, Netherlands. The Court's official seat is in The Hague, Netherlands, but its proceedings may take place anywhere. It can exercise its jurisdiction only when national courts are unwilling or unable to investigate or prosecute such crimes. I has 18 Judges: 18 elected for 9-year term. It has a membership of 122 countries while Israel, United States, Russia, china and India are not its member.

Relation with UN: While not a United Nations organization, the Court has a cooperation agreement with the United Nations. When a situation is not within the Court’s jurisdiction, the United Nations Security Council can refer the situation to the ICC granting it jurisdiction.

Hence both statement 1and 2 are correct.

Hence, option (c) is the correct answer.

Source: https://vajiramias.com/current-affairs/international-criminal-court- icc/5dce12961d5def077c485cf8/

Q80. The Sabarimala temple is located in which of the following Tiger Reserve? a. Parambikulam Tiger Reserve b. Periyar Tiger Reserve c. Anamalai Tiger Reserve d. Sathyamangalam Tiger Reserve

Answer: b

Explanation:

The Sabarimala temple is a temple complex located at Sabarimala inside the Periyar Tiger Reserve in Pathanamthitta district, Kerala. The temple is dedicated to the Hindu celibate deity Ayyappan also known as Dharma Sastha, who according to belief is the son of Shiva and Mohini, the feminine incarnation of Vishnu.

Hence, option (b) is the correct answer.

Source: https://vajiramias.com/current-affairs/sabarimala-verdict/5dce16021d5def0775ad50e3/

Q81. With reference to the Tobacco Board of India, consider the following statements:

1. It has started a novel initiative of Golden Leaf Awards for recognition of public service.

2. It is a statutory body established under Ministry of Commerce and Industry.

Which of the statements given above is/are correct? a. 1 only b. 2 only c. Both 1 and 2 VAJIRAM & RAVI d. Neither 1 nor 2

Answer: b

Explanation:

Statement 1 is incorrect:

The Golden Leaf Awards were created to recognize professional excellence and dedication in the tobacco industry by Tobacco Reporter, an international magazine in the year 2006. Tobacco Board of India has been awarded the Golden Leaf Award in the Most Impressive Public Service Initiative category for the year 2019, for its efforts to initiate various sustainability (green) initiatives in Flue-Cured Virginia (FCV) tobacco cultivation in India.

Statement 2 is correct:

Tobacco Board is a statutory body established under Ministry of Commerce and Industry. It is headquartered in Guntur, Andhra Pradesh. The main functions of the Board include regulating the production and curing of Virginia tobacco in India, improving the yields and quality of tobacco, facilitating sale of tobacco through e-auctions, undertaking various grower welfare measures and export promotion of tobacco.

Hence, option (b) is the correct answer.

Source: https://vajiramias.com/current-affairs/tobacco-board-of-india/5dce13891d5def077a0194d1/

Q82. The unique 'living root bridges' or ‘jingkiengjri’ are found in a. b. Nagaland c. Assam d. Sikkim

Answer: a

Explanation:

The jingkiengjri or living root bridges — aerial bridges built by weaving and manipulating the living roots of the Indian rubber tree — have been serving as connectors for generations in Meghalaya. Spanning between 15 and 250 feet and built over centuries, the bridges, primarily a means to cross streams and rivers, have also become world-famous tourist attractions. A root bridge uses traditional tribal knowledge to train roots of the Indian rubber tree, found in abundance in the area, to grow laterally across a stream bed, resulting in a living bridge of roots. The process begins with placing of young pliable aerial roots growing from Ficuselastica (India rubber) trees in hollowed out Areca catechu or native bamboo trunks. These provide essential nutrition and protection from the weather, and also perform as aerial root guidance systems. Over time, as the aerial roots increase in strength and thickness, the Areca catechu or native bamboo trunks are no longer required. VAJIRAM & RAVI

Hence, option (a) is the correct answer.

Source: https://vajiramias.com/current-affairs/jing-kieng-jri/5dda128e1d5def55a9bc8f06/

Q83. With reference to the Depositor Education and Awareness Fund (DEA Fund), consider the following statements:

1. The Banking Regulation Act, 1949 empowers the SEBI to establish the fund.

2. It envisages registration of institutions and grant of financial assistance to them for promotion of depositors’ awareness.

Which of the statements given above is/are correct? a. 1 only b. 2 only c. Both 1 and 2 d. Neither 1 nor 2

Answer: b

Explanation:

Section 26A in the Banking Regulation Act, 1949 empowers the RBI to establish Depositor Education and Awareness Fund (DEA Fund). Accordingly, a Scheme was formulated and notified in the Official Gazette In 2014. The Scheme envisages registration of institutions and grant of financial assistance to them for promotion of depositors’ awareness. With a view to widening and deepening depositor awareness efforts, it has now been decided to invite applications ‘on tap’ for registration of eligible entities. Hence only statement 2 is correct.

Hence, option (b) is the correct answer.

Source: https://vajiramias.com/current-affairs/depositor-education-and-awareness-fund-dea- fund/5dcf64e61d5def0776573002/

Q84. With reference to the secretagogin, consider the following statements:

1. It is a protein that in humans is encoded by the SCGN gene.

2. It is a secreted calcium-binding protein which is found in the cytoplasm.

Which of the statements given above is/are correct? a. 1 only b. 2 only c. Both 1 and 2 d. Neither 1 nor 2

Answer: c VAJIRAM & RAVI

Explanation:

Secretagogin is a protein that in humans is encoded by the SCGN gene. The encoded protein is a secreted calcium-binding protein which is found in the cytoplasm.

Scientists at the CSIR-Centre for Cellular and Molecular Biology (CCMB) in Hyderabad have recently demonstrated the role of a protein secretagogin (SCGN) in increasing insulin action in obesity-induced diabetes. CCMB said that the findings established SCGN as a functional insulin-binding protein with therapeutic potential against diabetes. Hence both statements are correct.

Hence, option (c) is the correct answer.

Source: https://vajiramias.com/current-affairs/secretagogin-scgn/5dd89faa1d5def55aa76823b/

Q85. Which of the following diseases is caused by Enterotoxigenic E. coli (ETEC) bacteria? a. Diarrhea b. Cholera c. Dengue d. Typhoid

Answer: a

Explanation:

Enterotoxigenic E. coli (ETEC) bacteria are a primary cause of diarrhea, leading to substantial illness and death in children in low- and middle-income countries (LMICs). Currently there is no ETEC vaccine available on the market for use in children in ETEC high-risk areas. Recently an oral ETEC vaccine candidate, named ETVAX, was developed at University of Gothenburg in collaboration with Scandinavian Biopharma, Stockholm. ETVAX consists of inactivated E. coli bacteria expressing high levels of protective antigens and the ETEC-based B subunit protein LCTBA.

Hence, option (a) is the correct answer.

Source: https://vajiramias.com/current-affairs/etvax-vaccine/5dd89f531d5def2fe03636d5/

Q86. With reference to the Press Council of India (PCI), consider the following statements:

1. It is a quasi-judicial authority which was first set up in the year 1948.

2. Its chairman should be a retired judge of the Supreme Court of India.

Which of the statements given above is/are correct? a. 1 only b. 2 only c. Both 1 and 2 d. Neither 1 nor 2 VAJIRAM & RAVI

Answer: b

Explanation:

Statement 1 is incorrect:

The National Press Day is observed on 16th November every year. It was on this day the Press Council of India (PCI) started functioning as a moral watchdog to ensure that the press maintains high standards and is not fettered by any influence or threats. The Press Council of India was first set up in the year 1966 by the Parliament on the recommendations of the First Press Commission. It is a statutory body functioning under the Press Council Act, 1978. Objective is preserving the freedom of the press and of maintaining and improving the standards of press in India. It is a quasi-judicial authority. It adjudicates the complaints against and by the press for violation of ethics and for violation of the freedom of the press respectively.

Statement 2 is correct:

The PCI consists of a chairman and 28 other members. It is headed by the Chairman, who should be a retired judge of the Supreme Court of India.

Hence, option (b) is the correct answer.

Source: https://vajiramias.com/current-affairs/national-awards-for-excellence-in-journalism- 2019/5dd0b0b01d5def7ae0e23483/

Q87. Which of the following countries has a dispute with South Korea on issue of the Dokdo islands? a. Japan b. China c. Vietnam d. North Korea

Answer: a

Explanation:

The Dokdo Islands are the center of a diplomatic dispute between South Korea and Japan that goes back more than 300 years. In recent years, the relationship between South Korea and Japan has deteriorated, given the territorial dispute over the Dokdo islands — known as Takeshima in Japan. They are also referred to as the Liancourt islands. They are a group of small islets in the Sea of Japan While South Korea controls them, the islands are claimed by Japan.

Hence, option (a) is the correct answer.

Source: https://vajiramias.com/current-affairs/general-security-of-military-information-agreement- gsomia/5dda0dd71d5def2fe0366349/

Q88. With reference to 'Agni-II', consider the following statements: VAJIRAM & RAVI

1. It is a versatile surface-to-surface medium-range nuclear-capable missile that can carry a nuclear payload of 1000 Kgs.

2. The missile, yet to be inducted to the armed forces has a strike range of 4000 kms.

Which of the statements given above is/are correct? a. 1 only b. 2 only c. Both 1 and 2 d. Neither 1 nor 2

Answer: a

Explanation:

'Agni-II', an intermediate-range ballistic missile (IRBM) has already been inducted into the armed forces. It is a versatile surface-to-surface medium-range nuclear-capable missile that can carry a nuclear payload of one thousand kilograms. The missile has a strike range of 2000 km. The range of the 21- metre long missile weighing 17 tonnes can also be increased to three thousand kilometres by reducing the payload. This was the first time that this missile was test-fired at night. Hence only statement 1 is correct.

Hence, option (a) is the correct answer.

Source: https://vajiramias.com/current-affairs/agni-ii/5dd0b1571d5def7ae4dee3c6/

Q89. With reference to the WHO insulin prequalification programme, consider the following statements:

1. It is the first ever insulin prequalification programme to expand access to life-saving treatment for diabetes.

2. The WHO Prequalification of Medicines Programme contributes to increasing access to critical medical products in low- and middle-income countries.

Which of the statements given above is/are correct? a. 1 only b. 2 only c. Both 1 and 2 d. Neither 1 nor 2

Answer: c

Explanation:

Statement 1 is correct: VAJIRAM & RAVI

The World Health Organization (WHO) launched the first-ever insulin prequalification programme to expand access to life-saving treatment for diabetes. The decision is part of a series of steps WHO will take to address the growing diabetes burden in all regions. WHO prequalification of insulin is expected to boost access by increasing the flow of quality-assured products on the international market.

Statement 2 is correct:

The WHO Prequalification of Medicines Programme contributes to increasing access to critical medical products in low- and middle-income countries. The programme does this by evaluating medical products developed by manufacturers to ensure their quality, in turn expanding the pool of available quality medicines. Evaluating and prequalifying health products then guides international procurement agencies, such as the Global Fund, Gavi, the Vaccine Alliance, and UNICEF, and increasingly countries to make bulk purchases of medicines vaccines at lower prices.

Hence, option (c) is the correct answer.

Source: https://vajiramias.com/current-affairs/who-insulin-prequalification- programme/5dd1fcde1d5def7ae6d18f38/

Q90. Which of the following mutts was/were established by Adi Shankara, a prominent Hindu saint?

1. Govardhan Mutt

2. Sharada Mutt

3. Jyotir Mutt

Select the correct answer using the code given below: a. 1 only b. 1 and 2 only c. 2 and 3 only d. 1, 2 and 3

Answer: d

Explanation:

The Govardhan Mutt of Puri, one of the four key mutts established by AdiShankara, the prominent Hindu saint. The three others — Sharada Mutt (at Sringeri in Karnataka), Dwarka Mutt (in Gujarat) and Jyotir Mutt (at Joshimath in Uttarakhand). The management of these mutts has been vested with the respective Shankaracharyas. Two upa-peethas were also established in Kashi and Kanchi.

Hence, option (d) is the correct answer.

Source: https://vajiramias.com/current-affairs/govardhan-mutt/5dd1fd971d5def7ae0e2dae2/

Q91. Which of the following best describes ‘Bhaona’, recently seen in the news? a. A mythology-based theatrical performance prevalent is Assam. VAJIRAM & RAVI b. A ritual art form popular in north Kerala. c. A style of Indian painting, practiced in the Mithila region. d. None of the above.

Answer: a

Explanation:

Bhaona, a mythology-based theatrical performance, is a traditional form of entertainment, with religious messages, prevalent is Assam, India. It is a creation of Sankardeva, written in the early sixteenth century. He created the form to convey religious messages to villagers through entertainment. The plays of bhaona are popularly known as AnkiyaNats and their staging is known as bhaona. Bhaona is generally staged at satras and namghars in Assam. A Bhaona, involving dialogues, songs and dances by performers in costumes and ornaments, usually involves 40-50 people, including those playing heavy drums and cymbals. The bhaonas are written in the Assamese and Brajavali languages.

Hence, option (a) is the correct answer.

Source: https://vajiramias.com/current-affairs/bhaona/5dd1ff5d1d5def7ae6d18f7b/

Q92. With reference to the Pradhan Mantri Matru Vandana Yojana (PMMVY), consider the following statements:

1. It is targeted at women who are delivering their first and second child only.

2. A cash amount of INR 6,000 is transferred to the bank account of the beneficiary in three instalments upon meeting certain conditions.

Which of the statements given above is/are correct? a. 1 only b. 2 only c. Both 1 and 2 d. Neither 1 nor 2

Answer: b

Explanation:

The Pradhan Mantri Matru Vandana Yojana (PMMVY) is a maternity benefit scheme. The scheme is a conditional cash transfer scheme for pregnant and lactating women. It is targeted only at women delivering their first child. A cash amount of INR 6,000 is transferred to the bank account of the beneficiary in three instalments upon meeting certain conditions including early registration of pregnancy, having at least one ante-natal check-up and registration of child birth. Hence only statement 2 is correct.

Hence, option (b) is the correct answer. VAJIRAM & RAVI

Source: https://vajiramias.com/current-affairs/pradhan-mantri-matru-vandana-yojana- pmmvy/5dd35f121d5def7ae212a8e4/

Q93. With reference to the Rajya Sabha, consider the following statements:

1. The first Bill passed by Rajya Sabha was The Andhra State Bill, 1953.

2. The first and the only time when a Presiding Officer of Rajya Sabha casted his vote was in 1991.

Which of the statements given above is/are correct? a. 1 only b. 2 only c. Both 1 and 2 d. Neither 1 nor 2

Answer: b

Explanation:

On the occasion of 250th session of Rajya Sabha, Chairman released a publication, titled “RajyaSabha: The Journey since 1952” providing glimpses of various aspects of functioning of the House.

Key highlights of the publication:

• First sitting of the House was held on 13.5.1952 • First Bill passed: The Indian Tariff (Second Amendment) Bill, 1952 • First Bill concerning social change: The Special Marriages Bill, 1952 • First Constitution Amendment Bill passed by RajyaSabha: The Constitution (Second Amendment) Bill, 1953 for readjustment of representation in LokSabha by increasing the size of population per constituency. • First on reorganization of States: The Andhra State Bill, 1953 • Members: Dr.Mahendra Prasad is serving the highest number of 7th term followed by Dr. serving 6th term. • The representation of women in RajyaSabha increased from 15 (6.94%) in 1952 increased to 31(12.76%) in 2014 and is now 26(10.83%) in 2019. • The first and the only time when a Presiding Officer of RajyaSabha cast his vote was in 1991 when the Panel Chairman Shri M.A.Baby did so when the voting was tied on the Statutory Resolution moved by the opposition seeking disapproval of the Code of Criminal Procedure (Amendment) Ordinance resulting in the victory of the opposition in the House. • President Rule approved only by RajyaSabha: It happened only twice in respect of extension of President Rule in Tamil Nadu and Nagaland in 1977 and in case of Haryana in 1991 when LokSabha was dissolved. VAJIRAM & RAVI

• Bill passed by RajyaSabha but negative by LokSabha: The Constitution (Sixty-fourth Amendment) Bill, 1990 seeking to amend Article 356 relating to extension of President’s Rule in Punjab. So, only statement 2 is correct.

Hence, option (b) is the correct answer.

Source: https://vajiramias.com/current-affairs/250th-session-of-rajya- sabha/5dd361851d5def7ae0e324d2/

Q94. With reference to the Nagrik Kartavya Paalan Abhiyan, consider the following statements:

1. It is an initiative of Union Ministry of Law and Justice.

2. It aims to create mass awareness about the Fundamental Duties as enshrined in Constitution.

Which of the statements given above is/are correct? a. 1 only b. 2 only c. Both 1 and 2 d. Neither 1 nor 2

Answer: b

Explanation:

On the occasion of ‘Constitution Day’ Union Ministry of Human Resource Development, launched the kartavya.ugc.ac.in portal as a part of year long Nagrik Kartavya Paalan Abhiyan being observed throughout the country. The portal will be used primarily for holding monthly essay competitions for students as well as other activities like quizzes, debates, poster making etc pertaining to Nagrik Kartavya Paalan Abhiyan. On the occasion of the 70th year of the adoption of the Indian Constitution, it has been decided by the Government to run a Nagrik Kartavya Palan Abhiyan from 26th November 2019 to 26th November 2020, to create mass awareness about the Fundamental Duties as enshrined in Constitution. Hence only statement 2 is correct.

Hence, option (b) is the correct answer.

Source: https://vajiramias.com/current-affairs/nagrik-kartavya-paalan- abhiyan/5ddde3641d5def35c67e330b/

Q95. Which of the following are the main recommendations of the Company Law Committee-2019, recently seen in the news?

1. Establishment of benches of the National Company Law Appellate Tribunal.

2. Retention of status-quo in case of the non-compoundable offences.

3. Providing for appeal against the orders of the Regional Directors before the National Company Law Tribunal after due examination VAJIRAM & RAVI

Select the correct answer using the code given below: a. 1 and 3 only b. 1 and 2 only c. 2 and 3 only d. 1, 2 and 3

Answer: d

Explanation:

The report of the Company Law Committee-2019 was recently presented to the Union Minister of Finance and Corporate Affairs by Injeti Srinivas, Secretary, Ministry of Corporate Affairs, who chaired the Committee. The Committee was constituted by the Ministry of Corporate Affairs in September, 2019 to further decriminalise the provisions of the Companies Act, 2013 based on their gravity and to take other concomitant measures to provide further Ease of Living for corporates in the country.

The main recommendations of the Committee are as follows:

• Re-categorising 23 offences out of the 66 remaining compoundable offences under the Act, to be dealt with in the in-house adjudication framework wherein these defaults would be subject to a penalty levied by an adjudicating officer. • In addition, the quantum of penalties recommended are lower than the quantum of fines presently provided in the Act. • Retention of status-quo in case of the non-compoundable offences. • Proposing benches of the National Company Law Appellate Tribunal; • Extending applicability of Section 446B (lower penalties for small companies and one person companies) to all provisions which attract monetary penalties and extending the benefit to producer companies and start-ups also; • Providing power to enhance the thresholds which trigger applicability of Corporate Social Responsibility provisions; • Providing for appeal against the orders of the Regional Directors before the NCLT after due examination; • Exempting certain private placement requirements for Qualified Institutional Placements (QIPs) after due consultation with SEBI;

Hence, option (d) is the correct answer.

Source: https://vajiramias.com/current-affairs/company-law-committee- 2019/5dd360491d5def7ae4dfe29e/

Q96. With reference to the Bharatiya Poshan Krishi Kosh (BPKK), consider the following statements:

1. It aims at reducing malnutrition through a multi-sectoral results-based framework across the country.

2. Ministry of Women and Child Development has collaborated with Google for this project. VAJIRAM & RAVI

Which of the statements given above is/are correct? a. 1 only b. 2 only c. Both 1 and 2 d. Neither 1 nor 2

Answer: a

Explanation:

Statement 1 is correct:

Bharatiya Poshan Krishi Kosh (BPKK) will be a repository of diverse crops across 128 agro-climatic zones in India for better nutritional outcomes. The Kosh aims at reducing malnutrition through a multi-sectoral results-based framework, including agriculture, among women and children across the country.

Statement 2 is incorrect:

Ministry of Women and Child Development has collaborated with Bill and Melinda Gates Foundation for this project. The Bharatiya Poshan Krishi Kosh seeks to promote healthy dietary practices and tackle under-nutrition in a sustainable manner.

Hence, option (a) is the correct answer.

Source: https://vajiramias.com/current-affairs/bharatiya-poshan-krishi-kosh- bpkk/5dd35e5a1d5def7ae4dfe27f/

Q97. Which of the following vitamins is provided by Golden rice, that is produced through genetic engineering? a. Vitamin A b. Vitamin K c. Vitamin C d. None of the above

Answer: a

Explanation:

Golden rice was developed almost two decades ago by biologists from Switzerland and Germany as a way to prevent vitamin A deficiency in the developing world. Lack of vitamin A is a leading cause of childhood blindness and can also make children more susceptible to death from other illness like measles.

Vitamin A is made from beta-carotene, which is found in carrots, spinach, sweet potatoes, and other vegetables. To create golden rice, scientists modified rice plants with beta-carotene genes from maize. VAJIRAM & RAVI

By doing this, rice plants started to produce the rich orange-coloured pigment. Then, the transgenic plants were donated to publicly funded research centres to develop their own versions of golden rice using local rice varieties. Since the inception of golden rice in the late 1990s, debates have raged over the transgenic crop, considered a genetically modified organism (GM), and it has struggled to gain approval. Moreover, stringent international regulations such as the Cartagena Protocol have stymied approval of many GM crops.

Hence, option (a) is the correct answer.

Source: https://vajiramias.com/current-affairs/golden-rice/5ddb68aa1d5def55a9bcc573/

Q98. What is ‘Avian botulism’, recently in the news? a. A serious neuromuscular illness of birds. b. A phenomenon in which birds tend to kill their own species. c. A drastic destruction of habitat and poaching of Critically Endangered birds. d. A process by which endangered birds are saved during natural disasters.

Answer: a

Explanation:

In November, 2019 nearly 10,000 of migratory birds were found dead mysteriously in the Sambhar Salt area. It might be due to avian botulism. Avian botulism is a serious neuromuscular illness of birds caused by a toxin that is produced by the bacterium Clostridium botulinum. Avian botulism has been recognised as a major cause of mortality in wild-birds since the 1900s.

Hence, option (a) is the correct answer.

Value Addition: The Sambhar , India's largest inland salt lake, is located 80 km southwest of the city of Jaipur, Rajasthan. It surrounds the historical . Sambhar Lake has been designated as a (recognized wetland of international importance). Source: https://vajiramias.com/current-affairs/avian-botulism/5dd4977b1d5def2fe2857dbe/

Q99. Which of the following water bodies is bordered by the West Bank? a. Mediterranean Sea b. Dead Sea c. Red Sea d. Black Sea

Answer: b

Explanation: VAJIRAM & RAVI

The West Bank is the name given to the territory that was captured by Jordan after the 1948 Arab-Israeli War. Israel snatched it back during the Six Day War of 1967, and has occupied it ever since. The West Bank is a landlocked territory near the Mediterranean coast of Western Asia, bordered by Jordan to the east and by the Green Line separating it and Israel on the south, west and north. The West Bank also contains a significant section of the western Dead Sea shore.

Hence, option (b) is the correct answer.

Source: https://vajiramias.com/current-affairs/west-bank/5dd5fa8f1d5def2fe4dae51c/

Q100. Consider the following rivers:

1. Mapusa

2. Koyna

3. Bhima

4. Tungabhadra

Which of the rivers given above is/are tributaries of river Mahadayi? a. 1 only b. 2, 3 and 4 VAJIRAM & RAVI c. 1, 2 and 4 d. 2 and 3 only

Answer: a

Explanation:

The Mhadei (or Mahadayi/Mahadeyi) River is described as the lifeline of the Indian state of Goa. It originates from a cluster of 30 springs at Bhimgad in the in the Belgaum district of Karnataka. Then it enters Goa and finally drains in Arabian Sea. The Mandovi and the Zuari are the two primary rivers in the state of Goa. Mandovi joins with the Zuari at a common point at CaboAguada, forming the Mormugao harbour. The two rivers are linked by Cumbarjuem Canal. Panaji, the state capital and Old Goa, the former capital of Goa, are both situated on the left bank of the Mandovi. The river Mapusa is a tributary of the Mandovi. Three large freshwater isles — Divar, Chorao and Vashee are present in the Mandovi near the town of Old Goa.

Venna River, , Dudhganga River, Bhima River Ghataprabha River, , , Musi River and Dindi River are the major tributaries of River .

Hence, option (a) is the correct answer.

Source: https://vajiramias.com/current-affairs/mahadeyi-river/5dd74daa1d5def55a9bc2e97/